SlideShare una empresa de Scribd logo
1 de 29
Marketing y dirección de empresas
14
MICROECONOMIA
 Beas Reátegui, Isabel
 Figueiredo Aguilar, Gianella
 Toledo Quispe, Maricruz
EJERCICIOS
1. En el análisis de un intercambio entre dos personas entre dos personas, suponga
que ambas tiene la misma preferencia. ¿Será la curva de contrato una línea recta?
Explique su respuesta. ¿Puede dar un ejemplo en él no lo sea?
2. Cite un ejemplo de las condiciones en las que la frontera de posibilidades de
producción podría no ser cóncava.
3. en monpsonista compra trabajo por un salario inferior al competitivo. ¿Qué tipo
de ineficiencia provocará esta utilización del poder nonopsinio? ¿En qué
cambiaría su respuesta si el monopolista en el mercado de productos?
4. Juana tiene 8 litros de bebidas refrescantes y 2 sándwiches. Bartolo, en cambio,
tiene 2 lito de bebidas refrescantes y 4 sándwiches. Con estas dotaciones, la relación
marginal de sustitución (RMS) de Juana de sándwiches por bebidas refrescantes en
tres y la de Bartolo es uno. Represente una caja de edgaworth para mostrar si esta
asignación de los recursos es eficiente. En caso afirmativo, explique por qué. En
caso negativo indique qué intercambio mejoraría el bienestar de las dos partes.
5. La empresa Acme produce x e y unidades de los bienes alfa y beta,
respectivamente.
a. Utilice una frontera de posibilidades de producción para explicar por qué la
disposición a producir una cantidad mayor o menor de alfa depende de la
relación marginal de transformación de beta en alfa.
b. Considere dos casos extremos de producción: (i) Acme produce inicialmente cero
unidades de alfa o (ii) produce inicialmente cero unidades de beta. Si siempre
trata de permanecer en su frontera de posibilidades de producción, describa las
posiciones iniciales de los casos (i) y (ii). ¿Qué ocurre cuando la empresa Acme
empieza a producir ambos bienes?
6. En nuestro análisis de las caja de producción de Edgeworth, suponga que un
nuevo invento hace que el proceso de rendimientos constantes d escala s
convierte crecientes. ¿Cómo afecta este cambio a la curva de contratos
correspondiente a la producción?
7. Suponga que el oro (G) y la lata (S) son mutuamente sustitutivas porque ambos
sirven para protegerse de las inflaciones. Suponga también que las ofertas de los
dos metales se mantienen fijas a corto plazo (Qg = 50 y Qs = 200) y que las
demandas de oro y plata vienen dadas por las siguientes ecuaciones .
Pg = 850 - Qg + 0,5Ps
Ps = 540 - Qs + 0,2Pg
a. ¿Cuáles son los precios de equilibrio del oro y la plata?
b. Suponga que un nuevo descubrimiento de oro eleva la cantidad ofrecida en 85
unidades. ¿Cómo afecta este descubrimiento tato al precio del oro como al de la
plata?
EJERCICIOS
1. Muchos consumidores consideran que las marcas conocidas son una señal de
calidad y pagan más por los productos de marca (por ejemplo, la aspirina, la aspirina
Bayer en lugar de una aspirina genérica o las verduras congeladas de marca en
lugar de las que llevan la marca del supermercado). ¿Puede ser la marca una señal
útil de calidad? ¿Por qué sí o por qué no?
2. Gabriel ha acabado sus estudios universitarios recientemente. Después de trabajar
seis meses en su nuevo empleo, finalmente ha ahorrado lo suficiente para
comprarse su primer automóvil.
a. Tiene muy poca información sobre la diferencias entre las marcas y modelos. ¿Cómo
puede utilizar las señales de mercado, la reputación o la estandarización para hacer
comparaciones?
b. Usted trabaja en le departamento de préstamos de un banco. Después de
seleccionar un automóvil Gabriel acude a usted en busca de un préstamo. Como se
ha licenciado recientemente no tiene un largo historial crediticio. A pesar de eso, l
banco lleva mucho tiempo financiando automóviles a personas que se han
licenciado recientemente. ¿Es útil esta información en el caso de Gabriel? En caso
afirmativo, ¿Cómo?
3. Una importante universidad prohíbe la calificación de suspenso. Defiende su
medida alegando que los estudiantes tienden a rendir por encima de la media
cuando no están sometidos a presiones de suspenso. La universidad declara que
quiere que todos los estudiantes reciban las calificaciones de sobresaliente y
notable. Si el objetivo es elevar las calificaciones globales a notable o a un nivel
superior, ¿ es buena esta política?. Analice el ejercicio en relación con el problema
de riesgo moral.
4. El profesor Jiménez acaba de ser contratado por el departamento de economía
de un importe universidad. El rector ha declarado que la universidad de ha
comprometido a dar una educación de alta calidad a sus estudiantes universitarios.
Transcurridos los dos primeros meses del semestre, el profesor todavía no ha
iniciado sus clases. Parece que está dedicándose por completo a las
investigaciones económicas y descuidando la enseñanza. Sostiene que su
investigación dará más prestigio al departamento y a la universidad. ¿Debería
permitirse que continúa dedicándose exclusivamente a la investigación? Analicé
este ejercicio en relación con el problema del principal y del agente.
5. Dada su fama de producir automóviles con insatisfactorios historiales de
reparaciones han ofrecido amplias garantías a los compradores (por ejemplo, una
garantía durante siete años de todas las piezas y la mano de obra relacionadas con
problemas mecánicos).
a. Dados sus conocimientos sobre el mercado de <<cachorros>>, ¿Por qué es
razonable esta política?
b. ¿Es probable que plantee un problema de riesgo moral? Explique su respuesta.
6. Para fomentar la competencia y el bienestar de los consumidores, las autoridades
prohíben la publicidad engañosa. ¿Cómo se fomenta la competencia con publicidad
verás? ¿Por qué sería menos competitivo un mercado si las empresas realizaran
publicidad engañosa?
7. Una compañía de seguros está considerando la posibilidad de crear tres tipos de
pólizas de seguros contra incendios: (i) cobertura completa, (ii) cobertura
completa por encima de los primeros 10.000 dólares de pérdidas y (iii) cobertura
del 90 por ciento de todas las pérdidas. ¿Qué póliza tiene más probabilidades de
planear problemas de riesgo moral?
8. El lector ha visto que la información asimétrica puede reducir la calidad media de
los productos que se venden en el mercado, ya que los productos de mala calidad
expulsan a los de buena calidad. En los mercados en los que predominan la
información asimétrica, ¿Estaría usted de acuerdo con cada una de las medidas
siguientes o discreparía? Explique brevemente su respuesta.
a. El Estado debería subvencionar a las organizaciones de consumidores.
b. El Estado debería imponer normas de calidad, por ejemplo, debería prohibir a las
empresas la venta de artículos de mala calidad.
c. El productor de un artículo de buena calidad probablemente querrá ofrecer una
garantía amplia.
d. El Estado debería obligar a todas las empresas ofrecer amplias garantías.
9. Dos vendedores de automóviles usados compiten uno al lado del otro en una
carretera principal. El primero, Automóviles Hernández, vende automóviles de
buena calidad que inspecciona minuciosamente y, si es necesario, repara. La
compra y la reparación de cada uno de los automóviles que vende le cuesta, en
promedio, 5.000 dólares solamente. Si los consumidores conocieran la calidad de
los automóviles usados que compran, pagarían encantados 10.000 dólares, en
promedio, por los que vende Hernández y sólo 7.000, en promedio, por los que
vende López. Desgraciadamente, los vendedores
son demasiado nuevos para tener reputación, por lo que los consumidores no
conocen la calidad de sus automóviles. Se imaginan, sin embargo que tienen un 50
por ciento de probabilidades de acabar comprando un automóvil de buena
calidad, cualquiera que sea el vendedor al que acudan y, por lo tanto, están
dispuestos a pagar 8.500 dólares, n promedio, por un automóvil.
Hernández tiene una idea: ofrecer una garantía total por todos los automóviles
que vende. Sabe que una garantía que dure Y años le costará 500Y dólares en
promedio y que si López trata de ofrecer esa misma garantía. ésta le costará
1.000Y dólares, en promedio.
a. Supongamos que Hernández ofrece una garantía de un año por todos los
automóviles que vende. ¿Será ésta una señal creíble de calidad? ¿Hará o n López la
misma oferta de tal manera que los consumidores puedan suponer correctamente
que los automóviles de Hernández son de buena calidad debido a la garantía y
que, por lo tanto, valen 10.000 dólares en promedio?
b. ¿Qué ocurre si Hernández ofrece una garantía de dos años por sus automóviles?
¿Será está una señal creíble de calidad? ¿Y si ofrece una garantía de tres años?
c. Si tuviera que aconsejar a Hernández, ¿Qué periodo de garantía le instaría a
ofrecer? Explique por qué.
10. El ingreso a corto plazo de una empresa viene dado por I= 10e - e.e, donde e es el
nivel de esfuerzo del trabajador representativo (se supone que todos son
idénticos). Un trabajador eligeel nivel de esfuerzo que maximiza su salario, una vez
descontado el esfuerzo w - e (se supone que el coste unitario del esfuerzo es 1).
Halle el nivel de esfuerzo y el nivel de beneficio (el ingreso menos el salario pagado)
correspondiente a cada uno de los siguientes sistemas salariales. Explique por qué
estas diferentes relaciones del principal y el agente generan distinto resultados.
a. w = 2 cuando e >/ 1; de lo contrario, w = 0.
b. w = I /2.
c. w = I/12.5.
EJERCICIOS
1. Algunas empresas se han instalado en la parte oeste de una ciudad después de que
la parte este fuera ocupada por viviendas monoparentales. Cada empresa produce
el mismo producto y emite como consecuencia humos nocivos que afectan
negativamente a los residentes de comunidad.
a. ¿Por qué existe una externalidad creada por las empresas?
b. ¿Cree usted que la negociación privada puede resolver el problema de la
externalidad? Explique su respuesta.
c. ¿Cómo podría averiguar la comunidad el nivel eficiente de calidad del aire?
2. Un programador informático presiona en contra de que los programas informáticos
tengan derechos de autor. Sostiene que todo el mundo debería beneficiarse de los
programas innovadores hechos para computadoras personales y que la exposición
a una amplia variedad de programas inspira a los programadores jóvenes y los lleva
a crear programas aún más innovadores. Considerando los beneficios sociales
marginales que se derivan posiblemente de su propuesta, ¿Está usted de acuerdo
con la postura del programador?
3. Suponga que los estudios científicos suministran la siguiente información sobre los
beneficios y los costes de las emisiones de dióxido:
Beneficios de la reducción de las emisiones:
BM = 400 - 10A
Costes de la reducción de las emisiones:
CM = 100 - 20A
Donde A es la cantidad reducida en millones de toneladas y os beneficios y los
costes se expresan en dólares por tonelada.
a. ¿Cuál es el nivel de reducción de las emisiones socialmente eficiente?
b. ¿Cuáles son el beneficio marginal y el coste marginal de reducción correspondiente
al nivel de reducción socialmente eficiente?
c. ¿Qué ocurre con los beneficios sociales netos (los beneficios menos los costes) si
reducimos un millón de toneladas más que el nivel eficiente? ¿Un millón menos?
d. ¿Por qué es socialmente eficiente igualar los beneficios marginales y los costes
marginales en lugar de reducir la contaminación hasta que los beneficios totales
sean iguales a los costes totales?
4. Cuatro empresas situadas en diferentes puntos de un río vierten distintos cantidades
de residuos en él. Estos afectan negativamente la calidaddel agua en al que nadan
los propietarios de viviendas que viven en sus márgenes. Estas personas pueden
construir piscinas para evitar nadar en el río y las empresas pueden construir
piscinas parar evitar nadar en el río y las empresas pueden comprar filtros que
eliminen las sustancias químicas perjudiciales que contienen los residuos que
vierten en el río. Como asesor de un organismo de planificación regional. ¿Cómo
compararía y contrataría las siguientes opciones para hacer frente al efecto
perjudicial de los vertidos?
a. Una tasa uniforme sobre los vertidos de las empresas situadas en el río.
b. Una norma uniforme sobre el nivel se residuos que puede verter cada empresa.
c. Un sistema de permisos transferibles de vertidos, en el que el nivel agregado de
vertidos sea fijo y todas las empresas reciban permisos idénticos.
5. Las investigaciones médicas han mostrado los efectos negativos que produce el
tabaco en la salud de los fumadores <<pasivos>>. Las tendencias sociales recientes
apuntan a una creciente intolerancia hacia el consumo de tabaco en zonas
públicas. Si usted fuma y desea seguir fumando a pesar del endurecimiento de la
legislación contra el tabaco, describa el efecto que produciría las siguientes
propuestas legislativas en su conducta. Como consecuencia de estos programas,
¿Se beneficia usted como fumador individual? ¿Se beneficia la sociedad en su
conjunto?
a. Se propone una ley que reduciría los niveles de alquitrán y nicotina de todos los
cigarros.
b. Se establece un impuesto sobre cada paquete de los cigarrillos.
c. Se establece un impuesto sobre cada paquete de los cigarrillos.
d. Los fumadores deben llevar siempre su permiso para fumar expendido por el
Estado.
6. Un apicultor vive al lado de un manzanar, cuyo dueño de beneficia de las abejas
porque cada colmena poliniza alrededor de un acre de manzanos. Sin embargo, el
dueño del manzanar no paga nada por este servicio, porque las abejas acuden al
manzanar sin que él tenga nada que hacer. Como no hay suficientes abejas para
polinizar todo el manzanar, su dueño debe completar la polinización por medios
artificial con un coste de 10 dólares por acre de árbol.
La apicultura tiene un coste marginal CM = 10 + 2Q, donde Q es el número de
colmenas. Cada colmena produce miel por valor de 20 dólares.
a. ¿Cuántas colmenas mantendrá el apicultor?
b. ¿Es económicamente eficiente este número de colmenas?
c. ¿Qué cambios harían que esta actividad fuera más eficiente?
7. Existen tres grupos en una comunidad. Sus curvas de demanda de televisión
pública en horas de programación, T, vienen todas por.
W1 = 150 $ - T
W2 = 200$ - 2T
W3 = 250$ - T
Supongamos que la televisión pública es un bien público puro que puede producirse con
un coste marginal constante de 200 $ dólares por hora.
a. ¿Cuál es el número eficiente de horas de televisión pública?
b. ¿Cuánta televisión pública suministraría un mercado privado competitivo?
8. Reconsidere el problema de recursos comunes del Ejemplo 18.5. Suponga que la
popularidad de los cangrejos de río continúa aumentando y que la curva de
demanda se desplaza de C = 0,401 - 0,0064F a C = 0,50 - 0,0064F. ¿Cómo afecta
este desplazamiento de la demanda a la captura afectiva de cangrejos, a la
captura eficiente y al coste social del acceso común? Pista: utilice las curvas de
coste social marginal y de coste privado del ejemplo.
9. El banco Georges, zona pesquera sumamente productiva de la aguas de Nueva
Inglaterra, puede dividirse en dos zonas en función de la cantidad de peces. La
zona 1 tiene una cantidad mayor por millas cuadrada, pero la pesca muestra
grandes rendimientos decreciente. Las capturas diarias (en toneladas) son en la zona
1. F1 = 200(X1) - 2(X1) (X1).
donde X1 es el número de barcos pesqueros que faenan en ella. La zona 2 tiene
menos peces por millas cuadrada, pero es mayor y los rendimientos decrecientes
no son un problema tan serio. Sus capturas diarias son. F2 = 100(X2) - (X2) (X2).
donde X2 es el número de barcos pesqueros que faenan en esa zona. La captura
marginal de pescado, CMF, de cada zona pude representarse de la forma siguiente:
CMF1 = 200 - 4(X1)
CMF2 = 100 - 2(X2)
Actualmente hay 100 barcos que tienen licencia para faenar en estas dos zonas. El
pescado se vende a 100 dólares la tonelada. El coste total (de capital y de
explotación) por barco es constante e igual a 1.000 dólares diarios. Responda a las
siguientes preguntas en relación con esta situación.
a. Si se autoriza a los barcos a pescar donde quieran sin restricción alguna por parte
del Estado, ¿cuántos pescarán en cada zona? ¿Cuál será el valor bruto de las
capturas?
b. Si el gobierno puede restringir el número de barcos, ¿Cuántos debe asignar cada
zona? ¿Cuál será el valor de las capturas? Suponga que el número total de barcos
sigue siendo el 100.
c. Si aumenta el número de pescadores que quieren comprar barcos y sumarse a la
flota pesquera, ¿debe concederles licencia un gobierno que desee maximizar el
valor neto de las capturas? ¿Por qué sí o por qué no?
RESPUESTAS DE ALGUNOS EJERCICIOS
CAPÍTULO 1
1. a. Falso. En Estados Unidos. las posibilidades de sustitución de unas regiones
geográficas por otras son escasas o nulas. Por ejemplo, un consumidor de los
Ángeles no viajará a Houston (Atlanta) o a Nueva York para comer simplemente
porque los precios de las hamburguesas sean más bajos en esas ciudades.
Asimismo, un McDonald's o un Burger King de Nueva York no puede ofrecer
hamburguesas en Los Ángeles. En otras palabras, una subida del precio de los
restaurantes de comida rápida de Nueva York no afectará ni la cantidad
demandada ni a la cantidad ofrecida de los Ángeles u otras partes del País.
d. Falso. Aunque es improbable que los consumidores recorran todo el país para
comprar ropa, los oferentes pueden transportar fácilmente la ropa de una parte del
país a otra. Por lo tanto. si los precios de la ropa fueran considerablemente más
altos en Atlanta que en los Ángeles, las empresas de ropa transportarían la ropa
Atlanta, lo que reduciría el precio en esa ciudad.
e. Falso. Aunque algunos consumidores sean fieles a la Coca-Cola o a la Pepsi, hay
muchos que sustituirían una por otra en función de las diferencias de precios. Por lo
tanto. hay un único mercado de bebidas de cola.
CAPÍTULO 2
1. En 1998, Qd = 3.244 - 283P y Qs = 1.944 + 207P. Con los nuevos mercados, Qd = Qd
+ 200 = 3.444 - 283P, Qs =Qs, 3.444 - 283P = 1.944 + 207P, 1.500 = 490P y P* =
3,06 dólares. A P*, Q* = 3.244(3,06) = 2.378,02. Por lo tanto, P = 3,06 dólares, Q =
2.378.
10. a) La demanda total es Q = 3.244 - 283P; la demanda interior es Qd = 1.700 - 107P;
restando la demanda de exportación Qe = 1.544 - 176. El precio inicial de
equilibrio del mercado (citado en el ejemplo) es P* = 2,65 dólares, Con una
disminución de la demanda de exportación de 40 por ciento la demanda toral se
convierte en Q =Qd + 0.6QE = 1.700 - 107P + 0.6(1.544 - 176P) = 2.626,4 - 212,6P. La
demandad es igual a la oferta. Por lo tanto,
2.626,4 -212,6P = 1.944 + 207P
682,4 = 419,6P
Por lo tanto, P = 682,4/419,6 = 1, 626 dólares, o sea, 1,63 dólares. A este precio, Q =
2.281. Sí los agricultores deberían preocuparse. Con esta disminución de la
cantidad y del precio, el ingreso pasa de 6.609 millones de dólares a 3.718
millones.
f. Si el gobierno de Estados Unidos apoya un precio de 3,50 dólares, el mercado no
se encuentra en equilibrio. A este precio, la demanda es igual a 1.700 - 107(3,50) =
1.325,5 y la oferta es 1.944 + 207(3,50) = 2.668,5. Hay un exceso de oferta (1.343)
que el gobierno debe comprar, lo que cuesta 3,50$(1.343) = 4.700,5 millones de
dólares.
11. a. En primer lugar, considerando la oferta de los países que no pertenecen a la
OPEP, Sc = Q* = 13. Si Es= 0,10 y P*= 18, Es = d (P*/Q*) implica que d = 0,07.
Sustituyendo d, Sc y P por sus valores en la ecuación de oferta, c = 11,74 + 0,07P.
Asimismo dado que Qd =23, Ed = -b (P*/Q*) = -0,05 y b = 0,06. Si sustituimos b
por este valor, Qd por 23 y P por 18 en la ecuación de demanda, significa que 23
= a -0,06(18), por lo que a = 24,08.
Por lo tanto, Qd = 24,08 - 0,06P.
g. Al igual que antes, Es = 0,4 y Ed = -0,4: Es = d (P*/Q*) y Ed = -b (P*/Q*), lo que
implica que 0,4 = d(18/13) y -0,4 = -b(18/23). por lo tanto, d = 0,29 y b = 0,51. A
continuación despejamos c y a: Sc = c + dPy Qd = a - bP, lo que implica que 13 =
c + (0,29)(18) y 23 = a - (0,51)(18). Por lo tanto, c = 7,78 y a = 32,18.
CAPÍTULO 3
a. Véase la figura 3(a), en la que B representa el número de paquetes de mantequilla y
M el de margarina.
b. La convexidad significa que la curva está <<combada hacia adentro>>. En este caso
las curvas de indiferencia no son <<estrictamente convexas>>, ya que son líneas
rectas.
Figura 3(a)
c) La restricción presupuestaria es Y= P BB + P M, 20 = 2B + MM, B = 10 – 0,5M. Dado que
Guille se muestra indiferente entre la mantequilla y la margarina y que el precio de la
mantequilla es más alto que el de la margarina, sólo comprará margarina.
6.
a) Véase a la figura 3(b), en la que A representa la cantidad de bebidas alcohólicas y N
representa la cantidad de bebidas no alcohólicas.
b) Cualquiera que sea la combinación de A y N, Juárez está dispuesto a renunciar a una
cantidad menor de A para conseguir alguna de N en comparación con Sanz. Por lo tanto, la
RMS entre A y N de Juárez es menor que la de Sanz. Las curvas de indiferencia de Juárez son
menos inclinadas que las de Sanz en cualquier punto del gráfico.
c) Para maximizar la satisfacción, cada consumidor debe consumir unas cantidades
tales que la RMS entre dos mercancías cualquiera sea igual a su relación de precios. Sus RMS
deben ser iguales porque se enfrentan a los mismos precios. Pero como tienen preferencias
distintas, consumirán cantidades diferentes de los dos bienes A y B.
8. En la figura 3(c), representamos en los ejes las millas que vuela, M, y todos los demás
bienes, B, expresados en dólares. La pendiente de la recta presupuestaria es –PM/PB. El
precio de las millas recorridas varia cuando varían estas, por lo que la restricción
presupuestaria tiene un vértice en 25.000 y 50.000 millas. Supongamos que PM es 1 dólar
por milla cuando <25.000 millas, PM = 0,75 dólares cuando 25.000< M< 50.000 y PM = 0,50
dólares cuando M>50.000. Supongamos también que PB =1 dólar. En ese caso, la
pendiente del primer segmento es -1, la del segundo es -0,75 y la del último es -0,5.
CAPITULO 4
1.
a) Cuando el precio experimenta una pequeña variación, es apropiada la siguiente
formula de la elasticidad- punto: EF = % Q/% P. En el caso de los chips de computadora,
EP= -2, y en el de las unidades de disco, EP= -1. Sea IT= PQ el ingreso antes de la variación
de los precios e (IT= PQ)2 el ingreso después de las variación de los precios. En el caso,
IT=IT2 – IT. En el caso de los chips de computadora, IT= -12%IT. En el caso de las unidades
de disco, IT= -1%IT.
b) Aunque conocemos la sensibilidad de la demanda a las variaciones del precio,
necesitamos conocer las cantidades y los precios de los productos para hallar los ingresos
totales derivados de las ventas.
5.
a) La curva de demanda es una línea recta que tiene una ordenada en el origen de
P=12 y una abscisa en el origen de Q=6 (ya que Q =6 – P/2).
b) Si no hubiera peaje, el precio P seria 0, por lo que Q=6.
c) Si el peaje es de 6 dólares, Q=3. La pérdida de excedente del consumidor es de la
diferencia entre el excedente del consumidor correspondiente a P=0(36) y el excedente del
consumidor correspondiente a P=3(9), o sea, 27.
10.
Si las variaciones del precio son pequeñas, seria adecuada la fórmula de la elasticidad-punto.
Pero en este caso el precio de los alimentos se duplica en 2 dólares a 4, por lo que debe
utilizarse la elasticidad-arco: Ep= ( Q/ P) P/Q). Sabemos que EP= -1, P=2, P=2 y
Q=5.000. Por lo tanto, si la renta no varía, podemos hallar Q: -1 = ( Q/2)(2+1)/(5.000+
Q/2)= ( _Q.3)(10.000+ Q). Observamos que Q=-
2.500: reduce su número de consumo de alimentos de 5.000 a 2.500 unidades.
a) Una devolución de impuestos de 5.000 dólares implica un aumento de
la renta de5.000 dólares. Para calcular la respuesta de la demanda a la devolución de
impuestos, utilizamos la definición de la elasticidad-arco con respecto a la renta: E1 = 0,5, I=
25.000, I= 5.000, Q= 2.500. Hallamos Q: 0,5= ( Q/5.000)(25.000+2.500)/(2.500+ Q/2).
Dado que Q=238, aumenta su consumo de alimentos de 2.500 unidades a 2.738.
b) En su curva final de indiferencia, decide consumir 2.738 unidades de
alimentos (lo que le supone un gasto de 10.952 dólares) y una cantidad de todos los demás
bienes por valor de 19.048 dólares. Al precio inicial de los alimentos de 2 dólares, esta
combinación le habrá costado 2.738.2$+19.048$= 24.524 dólares. Por lo tanto, habría
tenido 476 dólares más para gastar en alimentos o en otros bienes de consumo y habría
disfrutado de un bienestar mayor.
CAPITULO 4 – APENDICE
1. La primera función de utilidad puede representarse como una serie de líneas rectas; la
segunda como una serie de hipérbolas en el cuadrante positivo; y la tercera como una serie
de curvas en forma de L. La única función de utilidad que es estrictamente convexa es la
segunda.
3. La ecuación de Slutsky es dX/dPx =aX/aP*/ u=u* - X ( X/ I), donde el primer término
representa el efecto-sustituto y el segundo el efecto-renta. Como no existe sustitución
cuando varia el precio con ese tipo de función de utilidad, el efecto sustitución es cero.
CAPITULO 5
2. El cuadro 5 adjunto indica los 4 estados mutuamente excluyentes
7. Los consumidores que tengan la renta X* compraran simultáneamente un seguro
injusto y harán apuestas injustas. Son reacios a experimentar grandes pérdidas de renta,
pero amantas de obtener grandes ganancias.
El congreso aprueba el arancel El congreso no aprueba el arancel
Baja tasa de crecimiento Estado 1: Estado 2:
Lento crecimiento con arancel Lento crecimiento sin arancel
Elevada tasa de crecimiento Estado 3: Estado 4:
Rápido crecimiento con arancel Rápido crecimiento sin
arancel
CAPITULO 6
1.
a) El producto medio del trabajo, PM, es igual a Q/L. El producto marginal del trabajo,
PM, es igual a: Q/ L. El cuadro adjunto indica los cálculos relevantes.
b) En este proceso de producción, el trabajo muestra rendimientos decrecientes,
característicos de todas las funciones de producción en las que hay un factor fijo. Cada
unidad adicional de trabajo genera un aumento menor de la producción que la anterior.
c) El producto marginal del trabajo puede ser negativo cuando hay congestión en la
fábrica de sillas. A medida que aumenta el número de trabajadores que utilizan una
cantidad fija de capital, se molestan, por lo que disminuye la producción.
5. Si el producto marginal del trabajo (PM) es mayor que el producto (PMe), cada unidad
adicional de trabajo es más productiva que la media de todas las unidades anteriores.
Añadiendo la última unidad, aumenta la medida de todas ellas. El PMe es máximo cuando
la producción de la última es igual a la medida de todas las anteriores.
8.
a) Sea Q la producción de DISK, Inc., Q la producción de FLOPPY, Inc., y X las cantidades
idénticas de capital de trabajo de las dos empresas. En ese caso Q=10X X= 10X y Q=10X X =
10X = 10X. Como Q = Q, las dos empresas generan la misma producción con los mismos
factores.
b) Con una cantidad fija de capital de 9 máquinas, las funciones de producción se
convierten en Q=30L y Q = 37,37L. Considere el cuadro adjunto:
El producto marginal del trabajo en cada unidad de trabajo superior a 1 unidad es mayor en
el caso de DISK, Inc.
CAPITULO 7
3.
a) El coste total, CT, es igual al coste fijo, CF, más el coste variable, CV. Como la
franquicia, F, es una cantidad fija, los costes fijos de la empresa aumentan en la cuantía de la
misma. En ese caso, el coste medio, igual a (CF +CV)/Q, y el coste fijo medio igual a (CF/Q),
aumentan en la cuantía de la franquicia media (F/Q). El coste variable medio no resulta
afectado por la franquicia, como el coste marginal.
b) Cuando se establece un impuesto t, los costes variables aumentan es tQ. El coste
variable medio aumenta en t (el coste fijo es constante), al igual que el coste (total) medio.
Como el coste total aumenta en t con cada unidad adicional, el coste marginal aumenta en t.
4. Probablemente se refiere al beneficio contable; se trata del concepto que se utiliza
habitualmente en la mayoría de los análisis sobre los resultados financieros de la empresa.
En ese caso, el artículo indica que existe una diferencia considerable entre los beneficios
contables y los económicos. Sostiene que según el convenio colectivo en vigor, los
fabricantes de automóviles deben pagar a muchos trabajadores aun cuando no trabajen, lo
cual implica que sus salarios son IRECUPERABLES durante la vigencia del convenio. Los
beneficios contables se calcularían restando los salarios pagados, pero los beneficios
económicos no, puesto que hemos dicho que son costes irrecuperables. Por lo tanto, es
posible que los fabricantes de automóviles obtengan beneficios económicos por sus ventas,
aun cuan do experimentan pérdidas contables.
5. Si la empresa puede producir una silla con 4 horas de trabajo o 4 de maquinaria o
cualquier combinación, la isocuenta es una línea recta cuya pendiente es -1 y los puntos de
intersección con los ejes son K = 4 y L =4. La recta isocoste, CT = 22L + 110K, tiene una
pendiente de 1/5 y unos puntos de intersección con los ejes de K = CT/110 y L = CT/22. El
punto minimizado del coste es una solución de esquina, donde L = 4 y K = 0 y CT = 88.
8. La producción de gasolina exige destilar el crudo y refinarlo para convertirlo en
gasolina. Dado que el coste marginal de producción es constante hasta que se
alcanza el límite de la capacidad en el caso de ambos procesos, las curvas de coste
marginal tienen forma de L vista en el espejo. El coste marginal total, CM=CM+CM,
donde CM, es el coste marginal de destilar crudo hasta alcanzar el límite de la
capacidad, Q y CM es el coste marginal de refinar el crudo destiladohasta alcanzar el
límite de la capacidad, Q. Si la capacidad máxima de la unidad de destilación es
menor que la de la unidad de pirolisis, el CM total es vertical en Q (véase la figura 7).
Si la capacidad máxima de la unidad de pirolisis es menor que la de unidad de
destilación, el CM total es vertical en Q.
CAPITULO 7 - APENDICE
1.
a. El concepto de rendimientos de escala se refiere a la relación entre la producción y
los aumentos proporcionales de todos los factores. Si F (L, K) F (L, K), hay rendimientos
crecientes de escala; si F (L, K) =F (L, K), hay rendimientos constantes de escala; si F (L, K) F
(L, K), hay rendimientos decrecientes de escala. Aplicando esta definición a F (L, K) = KL, F (L,
K) = (K) (L) = KL = F (L, K) F (L, K). Por lo tanto, esta función de producción muestra
rendimientos crecientes de escala.
b. F (L, K) = 10K + 5L = F (L, K). la función de producción muestra rendimientos
constantes de escalas.
c. F (L, K) = (KL) = () = (KL) = (KL) = F (L, K). La función de producción muestra
rendimientos constates de escala.
2. El producto marginal del trabajo es 100K. El producto marginal del capital es 100L.
La relación marginal de sustitución técnica es K/L. Igualemos esta relación al
cociente entre el salario y la tasa de alquiler del capital: K/L = 30/120 o sea L = 4K. A
continuación sustituimos L en la función de producción por su valor y despejamos el
valor de K con el que se obtiene un nivel de producción de 1.000 unidades = 100K
4K. Por lo tanto, K = 2,5 y el coste total es igual a 379,20 dólares.
CAPITULO 8
1. El cuadro adjunto muestra la información sobre el ingreso y los costes de la empresa
cuando el precio baja a 35 dólares. A un precio de 35 dólares, la empresa debería producir
7 unidades para maximizar los beneficios
3.
a. Los beneficios se maximizan cuando el coste marginal (CM) es igual al ingreso marginal
(IM). En este caso, IM es igual a 60 dólares. Igualando CM a 60 se obtiene una cantidad
máximizadora de los beneficios de 30.
b. Los beneficios son iguales al ingreso total (PQ) menos el coste total. Por lo tanto, PQ –
100 – Q. Si P = 60 y Q = 30, beneficios = 800.
c. La empresa produce a corto plazo si sus ingresos son mayores que sus costes variables.
La curva de oferta a corto plazo de la empresa es el tramo de su curva CM situado por
encima del CVMe mínimo. En este caso, CVMe es el coste variable, Q2, dividido por la
cantidad, Q. Por lo tanto, CVMe = Q. Además, CM es igual a 2Q. Por lo tanto, CM es mayor
que CVMe cuando las cantidades son superiores a 0. Eso significa que la empresa produce
a corto plazo mientras el precio sea positivo.
5.
a. Cuando se establece un impuesto de 1 dólar sobre una única empresa, todas sus curvas
de costes se desplazan en sentido ascendente en 1 dólar.
b. Como la empresa es precio-aceptante, el establecimiento de un impuesto sobre una
empresa solamente no altera el precio de mercado. Dado que la curva de oferta a corto
plazo de la empresa es su curva de coste marginal (por encima del coste variable medio) y
que la curva de coste marginal se ha desplazado en sentido ascendente (o hacia adentro), la
empresa ofrece menos al mercado a todos los precios.
c. Si se establece un impuesto sobre una única empresa, esta quebrara a menos que
obtuviera unos beneficios económicos positivos antes de que se estableciera el impuesto.
CAPITULO 9
1.
a. En el equilibrio de libre mercado, L = L. Por lo tanto, w = 4 dólares y L = L = 40. Si el
salario mínimo es de 5 dólares, L = 50 y L = 30. El número de personas empleadas viene
dado por la demanda del trabajo. Por lo tanto, los empresarios contratan 30 millones de
trabajadores.
b. Con una subvención, la empresa solo paga w-1. La demanda de trabajo se convierte en L
= 80 – 10(w – 1). Por lo tanto, w = 4,5’ dólares y L = 45.
4.
a. Igualando la demanda y la oferta, 28 – 2P = 4 + 4P. P = 4 y Q = 20.
b. La reducción del 25 por ciento exigida por el nuevo programa de pago en especie
implicaría que los agricultores producen 15.000 millones de búshels. Para animarlos a
reducir la superficie cultivada, el gobierno debedarles 5.000 millones de búshels que venden
en el mercado. Como la oferta total del mercado sigue siendo de 20.000 millones de
búshels, el precio del mercado sigue siendo de 4 dólares por búshels. Los agricultores
ganan 20.000 millones de dólares (4 dólares multiplicados por 5.000 millones de búshels)
con el programa, mientras que los consumidores no resultan afectados.
c. Los contribuyentes sales ganando por que el gobierno no tiene que pagar para almacenar
el trigo durante un año y transportarlo a un país subdesarrollado. El programa de pago en
especie puede durar mientras duren las reservas de trigo. Pero supone que la tierra que deja
de cultivarse puede volver a cultivarse cuando se agoten las existencias. En caso contrario,
los consumidores podrán acabar pagando más por los productos derivados del trigo. Por
último los agricultores obtienen unos beneficios extraordinarios por que no tienen costes de
producción.
9. Las curvas de oferta y demanda de gas natural pueden obtenerse aproximadamente de
la manera siguiente: Q = 14 + 2P + 0,25P, Q = -5P + 3,75P. Si el precio del petróleo es de
12 dólares el barril, estas curvas se convierten en Q = 17 + 2P y Q = 45 – 5P. Igualando Q y
Q, 17 + 2P = 45 – 5P P = 4 dólares. A este precio, la cantidad de equilibrioes de 25 bpc. Si se
establece un precio máximo de 1 dólar, los productos ofrecerán 19 bpc y los consumidores
demandaran 40. Los consumidores demandan el área A – B = 57 – 3,6 = 53,4 miles de
millones de dólares en la figura. Los productores pierden A – C = -57-9=66 miles de
millones de dólares. La pérdida irrecuperablede eficiencia es igual a 53,4-66=12,6 miles de
millones de dólares.
13. No. El caso más claro es aquel en la que los mercados de trabajo son competitivos. Con
cualquiera de los dos tipos de impuestos, la diferencia entre la oferta y la demanda debe ser
igual en total al 12,4 por ciento del salario de pagado. Da lo mismo que el impuesto lo
paguen totalmente los trabajadores (lo que reemplazaría la curva de oferta efectiva un 12,4
por ciento) o los empresarios (lo que desplazaría la curva de demanda efectiva un 12,4 por
ciento). Lo mismo ocurre con cualquier combinación de los dos impuestos que sume 12,4
por ciento.
CAPITULO 10
2. Hay tres importantes factores (1) ¿En qué medida son similares los productos que ofrecen
los competidores de Caterpillar? Si son sustitutivos cercanos, una pequeña subida de precio
podría llevar a los clientes a pasarse a la competencia. (2) ¿Qué antigüedad tiene el parque
existente de tractores? Una subida del precio de un cinco por ciento provoca un descenso
menor de la demanda si el parque de tractores es más antiguo. (3) En su calidad de capital
que intervi esperada del e como factor en la producción agrícola, ¿Cuál es la rentabilidad
esperada del sector agrícola? Si la renta agrícola esperada están ion disminuyendo, una
subida de los precios de los tractores provoca un descenso mayor de la demanda que el
que se estimaría con la información sobre las ventas y los precios pasados.
4.
a. La producción óptima se halla igualando el ingreso marginal y el coste marginal. Si la
función de demanda es lineal, P=a-bQ (aquí, a=100 y b=0,01), IM=a-2bQ=100-2(0,01) Q.
Coste total =30,000+50Q, por lo que Q=2.500. Introduciendo este resultado en la función
de demanda, P=100-0,01. 2.500=75 centavos. Los beneficios totales son -30.000+50 2.500-
0,01(2.500)= -30.00+125.000-62.500, o sea, 325 dólares a la semana.
b. Suponga que inicialmente los consumidores deben pagar el impuesto. Como el precio
(incluido el impuesto) que estarían dispuestos a pagar no varía, la función de demanda
puede expresarse de la forma siguiente: P+t=100-0,01Q-t. Como el impuesto eleva el precio
de cada unidad, el ingreso total del monopolista disminuye en tQ y el ingreso marginal
disminuye en t: IM=100-0,02Q-t, donde t= 10 centavos. Para hallar el nivel de producción
maximizador de los beneficios con el impuesto, igualamos el ingreso marginal y el coste
marginal: 100-0,02Q-10=5, o sea Q=2.000 unidades. A partir de la función de demanda,
100-0,01 2.000-10=70 (2.000)-(30.000+50(2.000))=10.000 centavos, o sea, 100 dólares a la
semana.
8.
a. A favor: Aunque Alcoa controlaba alrededor de un 90 por ciento de la producción de
aluminio primario de Estados Unidos, la producción de aluminio secundario de la empresas
dedicadas al recicladorepresentaba un 30 por ciento de la oferta total de aluminio. Debería
ser posible que una proporción mucho mayor de la oferta de aluminio procediera de fuentes
secundarias. Por lo tanto, la elasticidad-preciode la demanda de aluminio primario de Alcoa
es mucho mayor de lo esperado. En muchas aplicaciones, otros metales, como el cobre o el
acero, son sustitutos viables del aluminio. En este caso, la elasticidad de la demanda a la que
se enfrenta Alcoa podría ser menor de lo esperado.
b. En Contra: La cantidad de oferta potencial es limitada. Por lo tanto, manteniendo un
precio alto y estable Alcoa podría obtener beneficios monopolísticos. Por otra parte, como
Alcoa producía inicialmente el metal que reaparecía reciclado, habría tenido en cuenta en las
decisiones de producción la influencia de la recogida en chatarra en los futuros precios. Por
lo Tanto, ejercía un control monopolístico efectivo en la oferta de metal secundario.
c. No se pidió a Alcoa que vendiera ninguna de sus instalaciones productivas de Estados
Unidos, sino que (1) se le prohibió que pujara por las dos plantas de aluminio primario
construidas por el Estado durante la Segunda Guerra Mundial, y (2) se le obligo a
deshacerse de su filial canadiense , que se convirtió en Alcoa.
11. No. En un mercado competitivo, una empresa considera que el precio es horizontal e
igual al ingreso medio, que es igual que el ingreso marginal. Si el costo marginal de la planta
b aumenta, el precio seguirá siendo igual al costo marginal, al coste marginal total y al
ingreso marginal de la A. Solo se produce la cantidad de la planta B (lo cual reduce, a su vez,
la cantidad total), como muestra la figura 10.
CAPITULO 11
1.
a. Obligando a los pasajeros a pasar, al menos, el sábado por la noche fuera de casa se
separa a los que viajan por motivos de negocios, que prefieren estar de vuelta el fin de
semana, de los turistas, que viajan el fin de semana.
b. Basando los precios en el lugar de residencia del comprador, se hace la selección
geográficamente. En este caso, los precios pueden reflejar los costes de transporte, que el
cliente paga independientemente de que el cemento se entregue en el lugar de residencia
del comprador o en la cementera.
c. Vendiendo procesadores de alimentos juntos con los vales de reembolso se divide a los
consumidores en dos grupos: (1) los consumidores que son menos sensibles al precio
(aquellos cuya demanda tiene una elasticidad menos) no piden el reembolso; y (2) los
clientes que son más sensibles al precio (aquellos cuya demanda tiene una elasticidad más
alta) solicitan el reembolso.
d. Una reducción temporal del precio del papel higiénico es un tipo de discriminación
intertemporal de precios. Los clientes sensibles al precio compran más papel durante el
periodo en que se reduce el precio, mientras que los clientes que no son más sensibles al
precio compran la misma cantidad.
e. El cirujano puede distinguir a los pacientes de renta alta de los pacientes de renta baja
negociando. El arbitraje no es un problema porque la cirugía plástica no puede transferirse
de los pacientes de renta baja a los de renta alta.
8. a. Un monopolista que tiene dos mercados debe elegir unas cantidades tales en cada uno
que los ingresos marginales sean idénticos en los dos e iguales al coste marginal. El coste
marginal es la pendiente de la curva de coste total, 30. Para hallar los ingresos marginales de
cada mercado, despejaremos el precio en función de la cantidad. A continuación
introducimos esta expresión del precio en la ecuación del ingreso total. PNY = 150–3QNY y PLA
= 120 – (3/2) QLA. Los ingresos totales son, pues, ITNY = QNYPNY = QNY (150-3QNY) e ITLA =
QLAPLA = QLA (120-(3/2)QLA). Los ingresos marginales son las pendientes de las curvas de
ingreso total: IMNY = 150-6QNY e IMLA = 120-3QLA.
A continuación, igualamos cada ingreso marginal con el coste marginal (=30), lo que implica
que QNY = 20 y QLA = 30. Con estas cantidades, hallamos el precio de cada mercado: PNY =
150-3.20=90 y PLA = 120-(3/2).30 = 75.
b. Con el nuevo satélite, Sal ya no puede separar los dos mercados. La función total de
demanda es la suma horizontal de los dos mercados. Si el precio es superior a 120, la
demanda total es simplemente la función de demanda de Nueva York. Si es inferior,
sumamos las dos demandas: QT= 50-(1/3) P + 80 – (2/3) P =130 –P. Sal maximiza los
beneficios eligiendo una cantidad total tal que IM=CM. El ingreso total es QP, donde P=130-
Q.IT = Q (130-Q), por lo que el ingreso marginal es 130-2Q. Igualando dicho ingreso
marginal y el coste marginal, se obtiene una cantidad maximizadora de los beneficios de 50
y un precio de 80. En el mercado de Nueva York, la cantidad es igual a 50-(1/3)80=231/3
y en
el de Los Ángeles, es igual a 80-(2/3)80 = 262/3
. En total, se compran 50 unidades a un precio
de 80.
c. En las condiciones de mercado de (a), los beneficios son iguales a la suma de los ingresos
de cada mercado menos el coste de producir la cantidad para ambos: QNYPNY + QLAPLA–
1.000-30(QNY+QLA) = 20.90+30.75-1.000+30(20+30) = 1.550. En las condiciones de mercado
de (b) los beneficios son iguales al ingreso total menos el coste de producir la cantidad para
ambos mercados: QP-(1.000+30Q)=50.80-(1.000+30.50)=1.500. Por lo tanto, Sal gana más
dinero cuando se separan los dos mercados.
En las condiciones de mercado de (a), en el mercado de Nueva York el excedente del
consumidor es (150-90).20 (1/2)=600 y en el de Los Ángeles es (120-75).30 (1/2)=675. En las
condiciones de mercados de (b), en el mercado de Nueva York el excedente del consumidor
es (150-80)231/3
(1/2)=817 y en el de Los Ángeles es (120-80).262/3
(1/2)=533. Los
neoyorquinos prefieren (b) porque el precio de equilibrio es 80 en lugar de 90, por lo que su
excedente del consumidor es mayor. Pero los clientes de Los Ángeles prefieren (a) porque el
precio de equilibrio es 75 en lugar de 80.
10. a. Si las demandas individuales son Q1 = 6-P, el excedente del consumidor individual es
igual a 18 dólares a la semana, o sea, 936 dólares al año. Una cuota de afiliación de 936
dólares captura todo el excedente del consumidor, incluso aunque no se cobre una tarifa por
utilizar las pistas, ya que el coste marginal es cero. Los beneficios semanales serian iguales al
número de tenistas serios, 1.000, multiplicadopor la cuota semanal de afiliación, 18 dólares,
menos 5.000 dólares, que son los costes fijos, o sea, 13.000 dólares semanales.
b. Cuando hay dos clases de clientes, el dueño del club maximiza los beneficios cobrando
una tarifa por utilizar las pistas superiores al coste marginal y cobrando una cuota de
afiliación igual al excedente restante del consumidor que tenga la demanda más baja: el
tenista esporádico. La cuota de afiliación, T, es igual al excedente del consumidor restante
una vez calculada la tarifa que ha de cobrarse por utilizar las pistas: T= (Q2 – 0) (6-P) (1/2),
donde Q2=3-(1/2) P, o sea, T= {3-(1/2) P} (6-P) (1/2)=9-3P + P2
/4. Las cuotas de afiliación
totales cobradas a todos los jugadores serian 2.000(9-3P + P2
/4). Los ingresos generados por
las cuotas de afiliación son iguales a P (Q1+Q2)= P{1.000(6-P)+1.000(3-P/2)}=9.000P-1.500P2
.
Entonces, IT=2.000{(9-3P + P2
/4) + 9.000P-1.500P2
=18.000+3.000P-1.000P2
. El coste
marginal es cero y el ingreso marginal viene dado por la pendiente de la curva de ingreso
total: IT/ P =3.000-2.000P. Igualando el ingreso marginal y el coste marginal, se obtiene un
precio de 1,50 dólares por hora. El ingreso total es igual a 20.250 dólares. El coste total es
igual a los costes fijos de 5.000 dólares. Por lo tanto, los beneficios son iguales a 15.250
dólares a la semana, cantidad superior a los 13.000 dólares semanales que se obtienen
cuando solo se hacen socios los tenistas profesionales.
c. Una cuota de afiliación de 18 dólares semanales solo atraería a los tenistas serios, los
ingresos totales serian 54.000 dólares y los beneficios de 49.000 dólares a la semana. Si hay
tenistas serios como esporádicos, las cuotas de afiliación serian iguales a 4.000 multiplicado
por el excedente del consumidor del tenista esporádico: T=4.000(9-3P+ P2
/4). Las tarifas de
la utilización de las pistas son P{(6-P)3.000+(3-P/2)1.000}=(21P-3,5P2
)1.000. Entonces IT {4(9-
3P + P2
/4)+ (21P-3,5P2
)} 1.000= (36+9P-2,5P2
)1.000. Igualando el ingreso marginal y el coste
marginal, el precio es de (9/5), o sea, de 1,80 dólares por hora. En este caso, el ingreso total
es igual a 44.100 dólares. El coste total es igual a los costes fijos de 5.000 dólares. Los
beneficios con una tarifa de dos tramos son de 39.100 dólares semanales, cantidad que es
inferior a los 49.000 que se obtienen con tenistas profesionales solamente. El dueño del club
debería fijar unas cuotas anuales de 936 dólares y obtener unos beneficios de 2.548 millones
de dólares al año.
11. La venta conjunta mixta suele ser la estrategia ideal cuando las demandas solo están
correlacionadas algo negativamente o cuando los costes marginales de producción son
significativos. Los cuadros adjuntos presentan los precios de reserva de los tres
consumidores y los beneficios generados por las tres estrategias.
Precio de reservas (dólares)
Para el bien 1 Para el bien 2 Total
Consumidor A 3,25$ 6,00$ 9,25$
Consumidor B 8,25 3,25 11,50
Consumidor C 10,00 10,00 20,00
Precio 1 Precio 2 Venta conjunta
Beneficio
Venta por separado 8,25 $ 6,00$ ____ 28,50$
Venta conjunta pura ____ ____ 9,25$ 27,75
Venta conjunta mixta 10,00 6,00 11,50 29,00
La estrategia que maximiza los beneficios consiste en vender cada artículo por separado.
15. a. Los precios y los beneficios óptimos correspondientes a cada estrategia son:
Precio 1 Precio 2 Venta conjunta
Beneficio
Venta por separado 40,00 $ 40,00$ ____ 240,00$
Venta conjunta pura ____ ____ 100,00$ 400,00
Venta conjunta mixta 59,95 59,95 100,00 319,90
La venta conjunta pura es superior a la mixta porque cuando los costes marginales son cero,
no hay razón alguna para impedir a ningún cliente que compre los dos bienes.
b. Con un coste marginal de 35 dólares, los precios y los beneficios óptimos son:
Precio 1 Precio 2 Venta conjunta
Beneficio
Venta por separado 90,00 $ 90,00$ ____ 110,00$
Venta conjunta pura ____ ____ 100,00$ 120,00$
Venta conjunta mixta 59,95 59,95 100,00 110,00$
La venta conjunta mixta sigue siendo superior a todas las demás estrategias.
CAPITULO 11 – APENDICE
Examinamos cada caso y comparamos los beneficios.
a. Las cantidades y los precios óptimosg sin mercado exterior de motores son QMot
= QMon = 2.000, PMot = 8.000 dólares y PMon = 18.000 dólares. En el caso de la
división de motores, IT= 2.000. 8.000 $ = 16 millones de dólares; CT = 2(2.000) +
16 millones=31 millones de dólares y π = 4 millones de dólares. Los beneficios
totales son iguales a 12 millones de dólares.
b. Las cantidades y los precios optimos con un mercado exterior de motores son
QMot=1.500, QMon=3.000, PMot=6.000 dólares y PMon=17.000 dólares. En el caso de
la división de motores, IT=1.500. 6.000 $=9 millones de dólares; CT=2(1.500)2
=4,5
millones de dólares y π=4,5 millones de dólares. En el caso de división de
montanje, IT=3.000. 17.000 $=51 millones de dólares; CT=(8.000 +
6.000)3.000=42 millones de dólares y π= 9 millones de dólares. Los beneficios
totales son iguales a 13,5 millones de dólares.
c. Las cantidades y los precios optimos con un mercado monopolizado de motores
son QMot=2.200, QMon=1.600, PMot =8.800 dólares y PMon =18.400 dólares; se
venden 600 motores en el mercado monopolizado por 9.400 dólares. En el caso
de la división de motores, IT=1.600. 8.800$ + 600. 9.400=19,72 millones de
dólares; CT=2(2.200)2
=9,68 millones de dólares y π= 10,04 millones de dólares.
En el caso de la división de montaje, IT=1.600. 18.400 $=29,44 millones de
dólares; CT=(8.000+8.800)1.600=26,88 millones de dólares y π=2,56 millones de
dólares. Los beneficios totales son iguales a 12,6 millones de dólares.
La división superior, que construye motores, obtiene unos beneficios máximos
cuando tiene el monopolio de los motores. La división inferior, que construye
automóviles, obtiene unos beneficios máximos cuando hay un mercado
competitivo de motores. Dado el elevado coste de los motores, la empresa
obtiene mejores resultados cuando los motores se producen con el menos coste
posible con un mercado competitivo exterior de motores.
CAPITULO 12
1. Cada empresa obtiene beneficios económicos distinguiendo su marca del
resto. Si estos competidores se fusionaran para formar una única empresa, el
monopolista resultante no producirá tantas marcas como antes de la fusión.
Pero producir varias marcas con precios y características distintas es una
manera de dividir el mercado en grupos de clientes con elasticidades-precio
diferentes.
2. a. Para maximizar los beneficios π=53Q – Q2
– 5Q, hallamos π/ Q = -
2Q+48=0. Q=24, por lo que P=29. Los beneficios son iguales a 576.
b. P=53-Q1-Q2π1=PQ1-C(Q1)=53Q1-Q1
2
-Q1Q2-5Q1 y π=PQ2-C(Q2)=53Q2-Q1Q2-
Q2
2-5Q2.
c. El problema al que se enfrenta la empresa 1 consiste en maximizar los
beneficios, dado que la produccion de la 2 no variara en respuesta a la
decision de produccion de la 1. Por lo tanto, la 1 elige el valor de Q1 que
maximice π1, al igual que antes. La variación de π1 con respecto a una
variación de Q1 es 53-2Q1–Q2-5=0, lo que implica que Q1=24- Q2/2. Dado que
el problema es simétrico, la función de reacción de la empresa 2 es Q2=24-
Q1/2.
d. Hallamos los valores de Q1 y Q2 que satisfacen ambas funciones de
reaccion: Q1=24-(1/2)(24-Q1/2). Por lo tanto, Q1=16 y Q2=16.
El precio es P=53-Q1-Q2=21. Los beneficios son π2=P.Q1-C (Qi)=256. Los beneficios totales
de la industria son π1+π2=512.
5. Verdadero. La curva de reacción de la empresa 2 es q2=7,5-1/2q1 y la curva de reacción
de la 1 es q1=15-1/2q2. Sustituyendo, tenemos que q2=0 y q1=15. El precio es 15, que es el
precio monopolístico.
11. a. Para hablar el equilibrio de Nash, calculamos la función de reacción de cada empresa y
hallamos simultáneamente el precio. Suponiendo que el coste marginal es cero, los
beneficios de la empresa 1 son P1Q1= P1 (20- P1+ P2 )=20 P1 – P2
1+P2P1.IM1=20-2P1+P2. Al
precio maximizadode los beneficios, IM1=0. Por lo tanto, P1 =(20 + P2)/2. Como las empresas
1 y 2 son simétricas, el precio maximizado de los beneficios de la 2 es P2=(20+P1)/2.
Introducimos la función de reacción de la empresa 2 en la de la 1: P1={20+(20+P1)/2}/2=15 +
P1/4. P1=20. Por simetría, P2=20. En ese caso, Q1=20 y por simetría Q2=20. Los beneficios de
la empresa 1 son P1Q1=400 y los de la 2 también es 400.
b. Si la empresa 1 es la primera en fijar su precio, tiene en cuenta la función de reacción de la
empresa 2. Los beneficios de la empresa 1 son π1=P1 {20- P1 +(20 + P1)/2}. En ese caso,
dπ1/dP1=20-2P1+10+ P1. Igualando esta expresión a cero, P1 =30. Sustituyendo P1 por su
valor en la función de reacción de la empresa 2, P2 =25. A estos precios, Q1=20-30+25=15 y
Q2=20+30-25=25. Los beneficios son π1=30.15=450 y π2=25.25=625.
c. Su primera opción debería ser la (iii) y la segunda debería ser la (ii). Fijar unos precios
superiores a los valores del equilibrio de Cournot es opcional para ambas empresas cuando
se siguen estrategias de Stackelberg. A partir de las funciones de reacción, sabemos que el
líder de precios induce al seguidor al subir el precio. Pero el seguidor sube el precio menos
que el líder y, por lo tanto, cobra un precio más bajo que este. Las dos empresas disfrutan de
mayores beneficios, pero la seguidora obtiene mejores resultados y las dos obtienen mejores
resultados que en el equilibrio de Cournot.
CAPITULO 13
1. Si los juegos se repiten indefinidamente y todos los jugadores conocen todas las
ganancias, la conducta racional conducirá a unos resultados aparentemente
colusorios. Pero a veces las ganancias de otras empresas solo pueden conocerse
realizando amplios intercambios de información.
Tal vez el mayor problema que plantea el mantenimiento de un resultado pactado
sean las variaciones exógenas de la demanda y de los precios de los factores. Cuando
los jugadores no disponen todos ellos simultáneamente de la nueva información, la
reacción racional de una empresa podría ser interpretada por otra como una
amenaza.
2. Puede surgir un exceso de capacidad en las industrias en las que es fácil entrar y hay
productos diferenciados. Como las curvas de demanda de pendiente negativa de
cada empresa dan lugar a unos niveles de producción cuyo coste medio es superior
al coste medio mínimo, los aumentos de la producción provocan una disminución del
coste medio. La diferencia entre la producción resultante y la producción
correspondiente al coste medio mínimo a largoplazoes el exceso de capacidad, que
puede utilizarse para disuadir a otras empresas de entrar.
3. a. Hay dos equilibrios de Nash, (100,800) y (900,600).
b. Ambos directivos elegirán una estrategia orientada al segmento superior y el
equilibrio resultante será (50,50), lo que generara menos beneficios a ambas partes.
c. El resultado cooperativo (900,600) maximiza los beneficios conjuntos de las dos
empresas.
d. La empresa 1 se beneficia extraordinariamente de la cooperación. En comparación
con la siguiente oportunidad mejor, la 1 se beneficia en 900-100=800, mientras que
la 2 pierde 800-600=200 con cooperación. Por lo tanto, la 1 necesitaría ofrecer a la
2200 como mínimo para compensarla por la perdida.
6. a. Sí, hay dos: (1) dado que la empresa 2 elige A, la empresa 1 elige C; dado que la
empresa 1 elige C, la 2 elige A. (2) Dado que la empresa 1 elige C, la 2 elige A. (2) Dado que
la empresa 2 elige C, la 1 elige A; dado que la empresa 1 elige A, la 2 elige C.
b. Si las dos empresas utilizan estrategias maximizan, la 1 elige el producto A y la 2 elige
el producto A y la 2 elige el producto A, por lo que ambas obtienen una ganancia de -10.
c. La empresa 2 elige el producto C con el fin de maximizar las ganancias en la castilla
10,20.
12. Aunque las subastas de antigüedades suelen tener componentes de valor privado, son
principalmente de valor común porque hay anticuarios. Nuestro anticuario está
decepcionadoen la subasta abierta al público de la ciudadcercana porque las estimaciones
del valor de las antigüedades varían mucho y ha sufrido la maldición del ganador. En su
ciudad natal, donde hay menos postores bien informados, la maldición del ganador no ha
sido un problema.
CAPITULO 14
1. Con este programa, la recta presupuestaria de los trabajadores es una línea recta en
un nivel de 5.000 dólares. No hay ningún incentivo para trabajar con este nuevo
programa. La oferta de trabajo solo es positiva cuando los salarios generan una renta
superior a 10.000 dólares.
4. La demanda del trabajo viene dada por el ingreso del producto marginal, IPML = IM.
OML. En el mercado competitivo, el precio es igual al ingreso marginal, por lo que
IM=10. El producto marginal del trabajo es igual a la pendiente de la función de
producción Q=12L – L2
. Está pendiente es igual a 12- 2L. La cantidad de trabajo que
maximiza los beneficios de la empresa se encuentra donde IMPL = ω, que es el
salario. Si ω=30, despejando L se obtiene 4,5 horas al día. Asimismo, si ω=60, se
obtiene un valor de L de 3 horas al día.
9. La renta económica es la diferencia entre los salarios totales de todos los
trabajadores ocupados menos la cantidad que habría incluido a estos trabajadores a
trabajar. Los salarios totales son iguales a ω lD=1.200 ω- 10ω2. La renta total que
habrían aceptado los trabajadores es el área situada debajo de la curva de oferta de
trabajo hasta la cantidad de trabajo demandada correspondiente a ω. A partir de la
función de oferta, sabemos que L=20 ω, o sea, ω=(1.200-10ω)/20. En ese caso, esta
área es un triángulo igual a LD. ωs.1/2=(1.200-10 ω){(1.200-10ω)/20}.1/2=36.000-
600ω + 2,5 ω2= -36.000=1.800-25ω. El máximo se encuentra en el punto en el que
está pendiente es igual a cero, o sea, ω=72. A un salario de 72 dólares, tienen trabajo
480 afiliados. Estos habrían estado dispuestos a trabajar por un renta total de 5.760
dólares (0,5.480.480/20). Reciben 34.560 dólares y disfrutan de unas rentas
económicas de 28.800 dólares (34.560-5.760).
CAPITULO 15
El valor actual descontado de los primeros 80 dólares pagados dentro de un año es
VAD=80/(1+0,10)1=72,73 dólares. El valor de todos estos cupones puede hallarse de la
misma forma: VAD=80{1/(1,10)1+1/(1,10)2+1/(1,10)3+1/(1,10)4+1/(1,10)5}=303,26 dolares.
El valor actual del pago final de 1,000 dólares en el sexto año es 1.000/1,16=564,47 dólares.
Por lo tanto, el valor actual de este bono es 303,26$ + 564,47$=867,73 dólares. Con un tipo
de interés del 15 por ciento, VAD=700,49 dólares.
Redefiniendo los términos, la ecuación del valor actual neto se convierte en VAN= -5-
5(1+R)-1 -1(1+R)-2 -0,5(1+R)-3 + 0,96{(1+R)-4+ (1+R)-5 + (1+R)-6 + (1+R)-7} + 0,96{(1+R)-
8 + (1+R)-9 + (1+R)-10 + (1+R)-11} + 0,96{(1+R)-12 + (1+R)-13 + (1+R)-14 + (1+R)-15} +
0,96{(1+R)-16 + (1+R)-17 + (1+R)-18 + (1+R)-19} + 0,96(1+R)-20 + 1(1+R)-20. Con un tipo
de interés del 4 por ciento, el VAN se convierte en -5-4,8075 -0,9246 -0,4445 + 3,0978 +
2,6482 + 2,2637 + 1,9349 + 0,4381 + 0,4564 = -320.000 dólares. La inversión no merece la
pena.
a. Si compramos una botella y la vendemos dentro de t años, pagamos 100 dólares ahora y
recibimos 100t0,5 cuando la vendamos. El VAN de esta inversión es VAN= 100+e-rt100t0,5.
Si compramos efectivamente una botella, elegimos el valor de t que maximice el VAN. La
condición necesaria es dVAN/dt= e-0,1t(50-t-0,5)-0,1e-0,1t.100t0,5=0. Resolviendo, se
obtiene que t=5. Si conservamos la botella durante 5 años, el VAN es -100+e-
0,1.5100.50.5=35,62. Como cada botella es una buena inversión, debemos comprar las 100.
b. Usted recibe 130 dólares hoy, pero pierde los 100.50, 5 dólares que recibiera vendiendo
las botellas dentro de cinco años. El VAN de la oferta es VAN=130-(e (0,1) (5)) (100) (5(0,5))=
-239<0. Por lo tanto, no debe venderlas.
c. Si el tipo de interés baja de 10 a 5 por ciento, el cálculo del VAN es el siguiente: VAN=-
100+E-0,05t.100t0, 5. Si conservamos la botella durante 10 años, el VAN máximo es -100+e-
0.05.10.100.100, 5=91,80 dólares.
9. a. Compare la compra del automóvil con el alquiler, suponiendo que r=0,04. El valor actual
del coste neto de la compra es igual a -15.000+6.000/(1+0,04)3=-9.666,02. El valor actual del
coste del alquiler es igual a -3.600/(1+0,04)-3.600/(1+0,04)2-3.600/(1+0.04)3=-9.990,33. Es
mejor comprar el automóvil si r=4 por ciento.
b. Compare una vez más la compra con el alquiler: -15.000+6.000/(1+0,12)3=-10.729,32 en
el caso de compra y -3.600/(1+0,12)-3.600/(1+0,12)2-3.600/(1+0,12)3=-8.646,6 en el caso de
alquiler. Es mejor alquiler el automóvil si r=12 por ciento.
c. A los consumidores les dará igual cuando el valor del coste de la compra y posterior venta
del automóvil se igual al valor actual del coste del alquiler: -15.000+6.000/(1+r)3
=-
3.600/(1+r)-3.600/(1+r)2
-3.600/(1+r)3
. Esto es cierto cuando r=4,96. El lector puede resolver
esta ecuación utilizando una calculadora o una hoja de cálculo o por aproximaciones
sucesivas.
CAPITULO 16
1. Incluso aunque las preferencias sean idénticas, la curva de contrato puede o no ser
una línea recta. Es fácil mostrarlo gráficamente. Por ejemplo, cuando ambas personas
tienen las funciones de utilidad U=x2
y, la relación marginal de sustitución viene dada
por 2y/x. No es difícil demostrar que las RMS de las dos personas son iguales en
todos los puntos de la curva de contrato y=(Y/X)/x, donde X e Y son las cantidades
totales de los dos bienes. Un ejemplo en el que la curva de contrato no es una línea
recta es aquel en el que las dos personas tienen rentas diferentes y un bien es
inferior.
2. La relación marginal de transformación es igual al cociente entre los costes
marginales de producir los dos bienes. La mayoría de las fronteras de posibilidades
de producción son “combadas hacia fuera”. Sin embargo, si los dos bienes se
producen con funciones de producción idénticas, la frontera de posibilidades de
producción es una línea recta.
3. El cambio del proceso de producción de rendimientos constantes de escala por uno
de rendimientos claramente crecientes no altera la forma de las isocuantas. Podemos
redefinir simplemente las cantidades correspondientes a cada isocuanta de tal forma
que los aumentos de la producción más que proporcionales. Partiendo de este
supuesto, la relación marginal de sustitución técnica no variaría, por lo que tampoco
variaría la curva de contrato de la producción.
CAPITULO 17
4. a. Hasta hace poco, los clientes pensaban que los automóviles americanos eran de
peor calidad. Para dar un giro a esta tendencia, las compañías americanas invirtieron
en control de la calidad, mejorandolos historiales potenciales de reparaciones de sus
productos. Señalaron la mejora de su calidad ofreciendo mejores garantías.
b. Existe riesgo moral cuando la parte asegurada (el propietario de un automóvil
americano que tiene amplia garantía) puede influir en la probabilidad o en la
magnitud del acontecimiento que desencadena el pago(la reparación del automóvil).
La cobertura de todas las piezas y la mano de obra en caso de problemas mecánicos
reduce el incentivo para cuidar el automóvil. Por lo tanto, las garantías amplias
plantean un problema de riesgo moral.
CAPITULO 18
5. Es necesario saber que el valor tiene para los propietarios de viviendas la posibilidad
de nadar en el rio, así como el coste marginal de reducción de la contaminación. La
elección de la política depende de los beneficios y los costes marginales de
reducción. Si se cobra a las empresas una tasa idéntica por los vertidos, estas los
reducirán hasta el punto en el que el coste marginal de reducción sea igual a la tasa.
Si esta reducción sea igual a la tasa. Si esta reducción no es suficiente para poder
nadar, podría elevarse la tasa.
El establecimiento de una norma solo es eficiente si las autoridades poseen una
información completa sobre los costes y los beneficios marginales de la reducción de
la contaminación. Por otra parte, la norma no animara a las empresas a reducir más
los vertidos si surgen nuevas tecnologías para filtrarlos. Un sistema de permisos
transferibles de contaminación también obliga a las autoridades a averiguar cuál es el
nivel de eficiente. Una vez distribuidos los permisos, surgirán un mercado y las
empresas cuyo coste de reducción de la contaminación sea más alto compraran
permisos a las empresas cuyo coste sea menor. Sin embargo, a menos que se vendan
inicialmente los permisos, no se obtendrá ningún ingreso.
6. a. Los beneficios se maximizan cuando el ingreso marginal es igual al coste marginal.
Con un ingreso marginal constante de 20 dólares y un coste marginal de 10+2Q,
Q=5.
B. Si las abejas no acuden al manzanar, el agricultor debe pagar 10 dólares por acre
por la polinización artificial. Como estaría dispuesto a pagar hasta 10 dólares al
apicultor para que mantuviera cada colmena adicional, el beneficio social marginal de
cada una es de 30 dólares, que es mayor que el beneficio privado marginal de 20
dólares. Igualando el beneficio social y marginal y el coste marginal, Q=10.
c. El cambio más radical que haría que esta actividad fuera más eficiente seria la
fusión del negocio del agricultor y el del apicultor. Esta fusión internalizaría la
externalidad positiva de la polinización de las abejas. Si no se produce la fusión, el
agricultor y el apicultor deberían firmar un contrato por los servicios de polinización.

Más contenido relacionado

La actualidad más candente

Ejercicios de microeconomia
Ejercicios de microeconomiaEjercicios de microeconomia
Ejercicios de microeconomiaAxelSanchezRamos
 
Capítulo 18 Las externalidades y los bienes públicos
Capítulo 18 Las externalidades y los bienes públicosCapítulo 18 Las externalidades y los bienes públicos
Capítulo 18 Las externalidades y los bienes públicosDannyMendoza1981
 
Ejemplos modelos econometricos
Ejemplos modelos econometricosEjemplos modelos econometricos
Ejemplos modelos econometricosSam Wilson
 
Ejercicios y problemas_resueltos_macroeconomia
Ejercicios y problemas_resueltos_macroeconomiaEjercicios y problemas_resueltos_macroeconomia
Ejercicios y problemas_resueltos_macroeconomiaDavid Frias
 
Excedente del consumidor y del productor
Excedente del consumidor y del productorExcedente del consumidor y del productor
Excedente del consumidor y del productorguadalupe martinez
 
Competencia monopolística y oligopolio
Competencia monopolística y oligopolioCompetencia monopolística y oligopolio
Competencia monopolística y oligopolioEduardo Basurto
 
Ejercicios aplicados integrales11
Ejercicios aplicados integrales11Ejercicios aplicados integrales11
Ejercicios aplicados integrales11freddy carrasco
 
Equilibrio general Introduccion Parte 1
Equilibrio general Introduccion Parte 1Equilibrio general Introduccion Parte 1
Equilibrio general Introduccion Parte 1Horacio Santander
 
Taller 2 microeconomía
Taller 2 microeconomíaTaller 2 microeconomía
Taller 2 microeconomíaAl Cougar
 
estudio de la demanda
estudio de la demandaestudio de la demanda
estudio de la demandatoretocz
 
Capítulo 11 la fijación de los precios con poder de mercado
Capítulo 11 la fijación de los precios con poder de mercadoCapítulo 11 la fijación de los precios con poder de mercado
Capítulo 11 la fijación de los precios con poder de mercadoDannyMendoza1981
 
Que es la estadistica
Que es la estadisticaQue es la estadistica
Que es la estadisticaDaday Rivas
 
Preguntas tipo test_macroeconomia_i_ade
Preguntas tipo test_macroeconomia_i_adePreguntas tipo test_macroeconomia_i_ade
Preguntas tipo test_macroeconomia_i_adeKatherine Flores
 
Estudio de los conceptos de la probabilidad
Estudio de los conceptos de la probabilidadEstudio de los conceptos de la probabilidad
Estudio de los conceptos de la probabilidadDaday Rivas
 
Ejercicios resueltos de_riesgo_y_rentabi
Ejercicios resueltos de_riesgo_y_rentabiEjercicios resueltos de_riesgo_y_rentabi
Ejercicios resueltos de_riesgo_y_rentabiRicardoLozadaCarbone
 
Examenes Resueltos de Macroeconomia
Examenes Resueltos de MacroeconomiaExamenes Resueltos de Macroeconomia
Examenes Resueltos de MacroeconomiaJose Matos
 

La actualidad más candente (20)

Ejercicios de microeconomia
Ejercicios de microeconomiaEjercicios de microeconomia
Ejercicios de microeconomia
 
Capítulo 18 Las externalidades y los bienes públicos
Capítulo 18 Las externalidades y los bienes públicosCapítulo 18 Las externalidades y los bienes públicos
Capítulo 18 Las externalidades y los bienes públicos
 
25 ejercicios de teoria y politica monetaria
25 ejercicios de teoria y politica monetaria25 ejercicios de teoria y politica monetaria
25 ejercicios de teoria y politica monetaria
 
Ejemplos modelos econometricos
Ejemplos modelos econometricosEjemplos modelos econometricos
Ejemplos modelos econometricos
 
Ejercicios y problemas_resueltos_macroeconomia
Ejercicios y problemas_resueltos_macroeconomiaEjercicios y problemas_resueltos_macroeconomia
Ejercicios y problemas_resueltos_macroeconomia
 
Excedente del consumidor y del productor
Excedente del consumidor y del productorExcedente del consumidor y del productor
Excedente del consumidor y del productor
 
Competencia monopolística y oligopolio
Competencia monopolística y oligopolioCompetencia monopolística y oligopolio
Competencia monopolística y oligopolio
 
Ejercicios aplicados integrales11
Ejercicios aplicados integrales11Ejercicios aplicados integrales11
Ejercicios aplicados integrales11
 
Equilibrio general Introduccion Parte 1
Equilibrio general Introduccion Parte 1Equilibrio general Introduccion Parte 1
Equilibrio general Introduccion Parte 1
 
Taller 2 microeconomía
Taller 2 microeconomíaTaller 2 microeconomía
Taller 2 microeconomía
 
estudio de la demanda
estudio de la demandaestudio de la demanda
estudio de la demanda
 
Capítulo 11 la fijación de los precios con poder de mercado
Capítulo 11 la fijación de los precios con poder de mercadoCapítulo 11 la fijación de los precios con poder de mercado
Capítulo 11 la fijación de los precios con poder de mercado
 
Microeconomía Cap. 1 El consumidor
Microeconomía Cap. 1 El consumidorMicroeconomía Cap. 1 El consumidor
Microeconomía Cap. 1 El consumidor
 
Que es la estadistica
Que es la estadisticaQue es la estadistica
Que es la estadistica
 
Econometría ii-cap 16 datos de panel
Econometría ii-cap 16 datos de panelEconometría ii-cap 16 datos de panel
Econometría ii-cap 16 datos de panel
 
Preguntas tipo test_macroeconomia_i_ade
Preguntas tipo test_macroeconomia_i_adePreguntas tipo test_macroeconomia_i_ade
Preguntas tipo test_macroeconomia_i_ade
 
Estudio de los conceptos de la probabilidad
Estudio de los conceptos de la probabilidadEstudio de los conceptos de la probabilidad
Estudio de los conceptos de la probabilidad
 
Ejercicios resueltos de_riesgo_y_rentabi
Ejercicios resueltos de_riesgo_y_rentabiEjercicios resueltos de_riesgo_y_rentabi
Ejercicios resueltos de_riesgo_y_rentabi
 
Examenes Resueltos de Macroeconomia
Examenes Resueltos de MacroeconomiaExamenes Resueltos de Macroeconomia
Examenes Resueltos de Macroeconomia
 
Resueltos
ResueltosResueltos
Resueltos
 

Destacado

Exp 4 costes de producción s
Exp 4 costes de producción sExp 4 costes de producción s
Exp 4 costes de producción sLuz Medina
 
Barreras de entrada y salida
Barreras de entrada y salidaBarreras de entrada y salida
Barreras de entrada y salidaMAIK8712
 
Capítulo 7 El coste de producción
Capítulo 7 El coste de producciónCapítulo 7 El coste de producción
Capítulo 7 El coste de producciónDannyMendoza1981
 
23 La Empresa La Competencia MonopolíStica
23 La Empresa La Competencia MonopolíStica23 La Empresa La Competencia MonopolíStica
23 La Empresa La Competencia MonopolíSticaCARLOS MASSUH
 
La inclusión educativa
La inclusión educativaLa inclusión educativa
La inclusión educativalosincreibles
 
Escuela inclusiva power point
Escuela inclusiva power pointEscuela inclusiva power point
Escuela inclusiva power pointTICSeinclusion
 
Educación Inclusiva
Educación InclusivaEducación Inclusiva
Educación Inclusivalasuveivi
 
INCLUSION EDUCATIVA
INCLUSION EDUCATIVAINCLUSION EDUCATIVA
INCLUSION EDUCATIVAberuscka
 
Escuela inclusiva
Escuela inclusivaEscuela inclusiva
Escuela inclusivaEva Monroy
 

Destacado (12)

Exp 4 costes de producción s
Exp 4 costes de producción sExp 4 costes de producción s
Exp 4 costes de producción s
 
Barreras de entrada y salida
Barreras de entrada y salidaBarreras de entrada y salida
Barreras de entrada y salida
 
Capítulo 7 El coste de producción
Capítulo 7 El coste de producciónCapítulo 7 El coste de producción
Capítulo 7 El coste de producción
 
23 La Empresa La Competencia MonopolíStica
23 La Empresa La Competencia MonopolíStica23 La Empresa La Competencia MonopolíStica
23 La Empresa La Competencia MonopolíStica
 
Cap 6 la produccion
Cap 6 la produccionCap 6 la produccion
Cap 6 la produccion
 
Educación inclusiva[1]diapositivas
Educación inclusiva[1]diapositivasEducación inclusiva[1]diapositivas
Educación inclusiva[1]diapositivas
 
La inclusión educativa
La inclusión educativaLa inclusión educativa
La inclusión educativa
 
Escuela inclusiva power point
Escuela inclusiva power pointEscuela inclusiva power point
Escuela inclusiva power point
 
Educación Inclusiva
Educación InclusivaEducación Inclusiva
Educación Inclusiva
 
INCLUSION EDUCATIVA
INCLUSION EDUCATIVAINCLUSION EDUCATIVA
INCLUSION EDUCATIVA
 
Ejercicios resueltos competencia perfecta
Ejercicios resueltos competencia perfectaEjercicios resueltos competencia perfecta
Ejercicios resueltos competencia perfecta
 
Escuela inclusiva
Escuela inclusivaEscuela inclusiva
Escuela inclusiva
 

Similar a Ejercicios Microeconomia

Similar a Ejercicios Microeconomia (20)

Chevrolet
ChevroletChevrolet
Chevrolet
 
Trabajo1 ipvc
Trabajo1 ipvcTrabajo1 ipvc
Trabajo1 ipvc
 
Analisis de decisiones 2 2012 tm
Analisis de decisiones 2 2012 tmAnalisis de decisiones 2 2012 tm
Analisis de decisiones 2 2012 tm
 
Supercauchos Vizcaya
Supercauchos VizcayaSupercauchos Vizcaya
Supercauchos Vizcaya
 
Analisis de decisiones 2 tm
Analisis de decisiones 2 tmAnalisis de decisiones 2 tm
Analisis de decisiones 2 tm
 
Preguntas y problemas de estudio
Preguntas y problemas de estudioPreguntas y problemas de estudio
Preguntas y problemas de estudio
 
Analisis de-sensibilidad
Analisis de-sensibilidadAnalisis de-sensibilidad
Analisis de-sensibilidad
 
Oligopolio
OligopolioOligopolio
Oligopolio
 
Control 2 Evaluacion 0910
Control 2 Evaluacion 0910Control 2 Evaluacion 0910
Control 2 Evaluacion 0910
 
Trabajo de economiaaaaaaaaaaaaaaaaaa
Trabajo de economiaaaaaaaaaaaaaaaaaaTrabajo de economiaaaaaaaaaaaaaaaaaa
Trabajo de economiaaaaaaaaaaaaaaaaaa
 
El riesgo financiero- Semana 5
El riesgo financiero- Semana 5El riesgo financiero- Semana 5
El riesgo financiero- Semana 5
 
Analisis de decisiones 2 2012 tm
Analisis de decisiones 2 2012 tmAnalisis de decisiones 2 2012 tm
Analisis de decisiones 2 2012 tm
 
PROBLEMAS DE ARBOL DE DECISIONES
PROBLEMAS DE ARBOL DE DECISIONESPROBLEMAS DE ARBOL DE DECISIONES
PROBLEMAS DE ARBOL DE DECISIONES
 
2101010220102El trailero demente
 2101010220102El trailero demente 2101010220102El trailero demente
2101010220102El trailero demente
 
Tuautoya
TuautoyaTuautoya
Tuautoya
 
Tuautoya
TuautoyaTuautoya
Tuautoya
 
Tuautoya
TuautoyaTuautoya
Tuautoya
 
Tuautoya
TuautoyaTuautoya
Tuautoya
 
Informacionasimetrica
InformacionasimetricaInformacionasimetrica
Informacionasimetrica
 
Examen PAU Economia 6
Examen PAU Economia 6Examen PAU Economia 6
Examen PAU Economia 6
 

Último

2 REGLAMENTO RM 0912-2024 DE MODALIDADES DE GRADUACIÓN_.pptx
2 REGLAMENTO RM 0912-2024 DE MODALIDADES DE GRADUACIÓN_.pptx2 REGLAMENTO RM 0912-2024 DE MODALIDADES DE GRADUACIÓN_.pptx
2 REGLAMENTO RM 0912-2024 DE MODALIDADES DE GRADUACIÓN_.pptxRigoTito
 
Tema 17. Biología de los microorganismos 2024
Tema 17. Biología de los microorganismos 2024Tema 17. Biología de los microorganismos 2024
Tema 17. Biología de los microorganismos 2024IES Vicent Andres Estelles
 
ACRÓNIMO DE PARÍS PARA SU OLIMPIADA 2024. Por JAVIER SOLIS NOYOLA
ACRÓNIMO DE PARÍS PARA SU OLIMPIADA 2024. Por JAVIER SOLIS NOYOLAACRÓNIMO DE PARÍS PARA SU OLIMPIADA 2024. Por JAVIER SOLIS NOYOLA
ACRÓNIMO DE PARÍS PARA SU OLIMPIADA 2024. Por JAVIER SOLIS NOYOLAJAVIER SOLIS NOYOLA
 
SESION DE PERSONAL SOCIAL. La convivencia en familia 22-04-24 -.doc
SESION DE PERSONAL SOCIAL.  La convivencia en familia 22-04-24  -.docSESION DE PERSONAL SOCIAL.  La convivencia en familia 22-04-24  -.doc
SESION DE PERSONAL SOCIAL. La convivencia en familia 22-04-24 -.docRodneyFrankCUADROSMI
 
PINTURA DEL RENACIMIENTO EN ESPAÑA (SIGLO XVI).ppt
PINTURA DEL RENACIMIENTO EN ESPAÑA (SIGLO XVI).pptPINTURA DEL RENACIMIENTO EN ESPAÑA (SIGLO XVI).ppt
PINTURA DEL RENACIMIENTO EN ESPAÑA (SIGLO XVI).pptAlberto Rubio
 
semana 4 9NO Estudios sociales.pptxnnnn
semana 4  9NO Estudios sociales.pptxnnnnsemana 4  9NO Estudios sociales.pptxnnnn
semana 4 9NO Estudios sociales.pptxnnnnlitzyleovaldivieso
 
Procedimientos para la planificación en los Centros Educativos tipo V ( multi...
Procedimientos para la planificación en los Centros Educativos tipo V ( multi...Procedimientos para la planificación en los Centros Educativos tipo V ( multi...
Procedimientos para la planificación en los Centros Educativos tipo V ( multi...Katherine Concepcion Gonzalez
 
OCTAVO SEGUNDO PERIODO. EMPRENDIEMIENTO VS
OCTAVO SEGUNDO PERIODO. EMPRENDIEMIENTO VSOCTAVO SEGUNDO PERIODO. EMPRENDIEMIENTO VS
OCTAVO SEGUNDO PERIODO. EMPRENDIEMIENTO VSYadi Campos
 
Abril 2024 - Maestra Jardinera Ediba.pdf
Abril 2024 -  Maestra Jardinera Ediba.pdfAbril 2024 -  Maestra Jardinera Ediba.pdf
Abril 2024 - Maestra Jardinera Ediba.pdfValeriaCorrea29
 
FUERZA Y MOVIMIENTO ciencias cuarto basico.ppt
FUERZA Y MOVIMIENTO ciencias cuarto basico.pptFUERZA Y MOVIMIENTO ciencias cuarto basico.ppt
FUERZA Y MOVIMIENTO ciencias cuarto basico.pptNancyMoreiraMora1
 
TEMA 14.DERIVACIONES ECONÓMICAS, SOCIALES Y POLÍTICAS DEL PROCESO DE INTEGRAC...
TEMA 14.DERIVACIONES ECONÓMICAS, SOCIALES Y POLÍTICAS DEL PROCESO DE INTEGRAC...TEMA 14.DERIVACIONES ECONÓMICAS, SOCIALES Y POLÍTICAS DEL PROCESO DE INTEGRAC...
TEMA 14.DERIVACIONES ECONÓMICAS, SOCIALES Y POLÍTICAS DEL PROCESO DE INTEGRAC...jlorentemartos
 
TALLER DE DEMOCRACIA Y GOBIERNO ESCOLAR-COMPETENCIAS N°3.docx
TALLER DE DEMOCRACIA Y GOBIERNO ESCOLAR-COMPETENCIAS N°3.docxTALLER DE DEMOCRACIA Y GOBIERNO ESCOLAR-COMPETENCIAS N°3.docx
TALLER DE DEMOCRACIA Y GOBIERNO ESCOLAR-COMPETENCIAS N°3.docxNadiaMartnez11
 
Tema 10. Dinámica y funciones de la Atmosfera 2024
Tema 10. Dinámica y funciones de la Atmosfera 2024Tema 10. Dinámica y funciones de la Atmosfera 2024
Tema 10. Dinámica y funciones de la Atmosfera 2024IES Vicent Andres Estelles
 
Prueba de evaluación Geografía e Historia Comunidad de Madrid 2º de la ESO
Prueba de evaluación Geografía e Historia Comunidad de Madrid 2º de la ESOPrueba de evaluación Geografía e Historia Comunidad de Madrid 2º de la ESO
Prueba de evaluación Geografía e Historia Comunidad de Madrid 2º de la ESOluismii249
 
PLAN DE REFUERZO ESCOLAR MERC 2024-2.docx
PLAN DE REFUERZO ESCOLAR MERC 2024-2.docxPLAN DE REFUERZO ESCOLAR MERC 2024-2.docx
PLAN DE REFUERZO ESCOLAR MERC 2024-2.docxiemerc2024
 
Análisis de los Factores Externos de la Organización.
Análisis de los Factores Externos de la Organización.Análisis de los Factores Externos de la Organización.
Análisis de los Factores Externos de la Organización.JonathanCovena1
 
Proyecto de aprendizaje dia de la madre MINT.pdf
Proyecto de aprendizaje dia de la madre MINT.pdfProyecto de aprendizaje dia de la madre MINT.pdf
Proyecto de aprendizaje dia de la madre MINT.pdfpatriciaines1993
 
CONCURSO NACIONAL JOSE MARIA ARGUEDAS.pptx
CONCURSO NACIONAL JOSE MARIA ARGUEDAS.pptxCONCURSO NACIONAL JOSE MARIA ARGUEDAS.pptx
CONCURSO NACIONAL JOSE MARIA ARGUEDAS.pptxroberthirigoinvasque
 
SISTEMA RESPIRATORIO PARA NIÑOS PRIMARIA
SISTEMA RESPIRATORIO PARA NIÑOS PRIMARIASISTEMA RESPIRATORIO PARA NIÑOS PRIMARIA
SISTEMA RESPIRATORIO PARA NIÑOS PRIMARIAFabiolaGarcia751855
 

Último (20)

2 REGLAMENTO RM 0912-2024 DE MODALIDADES DE GRADUACIÓN_.pptx
2 REGLAMENTO RM 0912-2024 DE MODALIDADES DE GRADUACIÓN_.pptx2 REGLAMENTO RM 0912-2024 DE MODALIDADES DE GRADUACIÓN_.pptx
2 REGLAMENTO RM 0912-2024 DE MODALIDADES DE GRADUACIÓN_.pptx
 
Tema 17. Biología de los microorganismos 2024
Tema 17. Biología de los microorganismos 2024Tema 17. Biología de los microorganismos 2024
Tema 17. Biología de los microorganismos 2024
 
Power Point: Fe contra todo pronóstico.pptx
Power Point: Fe contra todo pronóstico.pptxPower Point: Fe contra todo pronóstico.pptx
Power Point: Fe contra todo pronóstico.pptx
 
ACRÓNIMO DE PARÍS PARA SU OLIMPIADA 2024. Por JAVIER SOLIS NOYOLA
ACRÓNIMO DE PARÍS PARA SU OLIMPIADA 2024. Por JAVIER SOLIS NOYOLAACRÓNIMO DE PARÍS PARA SU OLIMPIADA 2024. Por JAVIER SOLIS NOYOLA
ACRÓNIMO DE PARÍS PARA SU OLIMPIADA 2024. Por JAVIER SOLIS NOYOLA
 
SESION DE PERSONAL SOCIAL. La convivencia en familia 22-04-24 -.doc
SESION DE PERSONAL SOCIAL.  La convivencia en familia 22-04-24  -.docSESION DE PERSONAL SOCIAL.  La convivencia en familia 22-04-24  -.doc
SESION DE PERSONAL SOCIAL. La convivencia en familia 22-04-24 -.doc
 
PINTURA DEL RENACIMIENTO EN ESPAÑA (SIGLO XVI).ppt
PINTURA DEL RENACIMIENTO EN ESPAÑA (SIGLO XVI).pptPINTURA DEL RENACIMIENTO EN ESPAÑA (SIGLO XVI).ppt
PINTURA DEL RENACIMIENTO EN ESPAÑA (SIGLO XVI).ppt
 
semana 4 9NO Estudios sociales.pptxnnnn
semana 4  9NO Estudios sociales.pptxnnnnsemana 4  9NO Estudios sociales.pptxnnnn
semana 4 9NO Estudios sociales.pptxnnnn
 
Procedimientos para la planificación en los Centros Educativos tipo V ( multi...
Procedimientos para la planificación en los Centros Educativos tipo V ( multi...Procedimientos para la planificación en los Centros Educativos tipo V ( multi...
Procedimientos para la planificación en los Centros Educativos tipo V ( multi...
 
OCTAVO SEGUNDO PERIODO. EMPRENDIEMIENTO VS
OCTAVO SEGUNDO PERIODO. EMPRENDIEMIENTO VSOCTAVO SEGUNDO PERIODO. EMPRENDIEMIENTO VS
OCTAVO SEGUNDO PERIODO. EMPRENDIEMIENTO VS
 
Abril 2024 - Maestra Jardinera Ediba.pdf
Abril 2024 -  Maestra Jardinera Ediba.pdfAbril 2024 -  Maestra Jardinera Ediba.pdf
Abril 2024 - Maestra Jardinera Ediba.pdf
 
FUERZA Y MOVIMIENTO ciencias cuarto basico.ppt
FUERZA Y MOVIMIENTO ciencias cuarto basico.pptFUERZA Y MOVIMIENTO ciencias cuarto basico.ppt
FUERZA Y MOVIMIENTO ciencias cuarto basico.ppt
 
TEMA 14.DERIVACIONES ECONÓMICAS, SOCIALES Y POLÍTICAS DEL PROCESO DE INTEGRAC...
TEMA 14.DERIVACIONES ECONÓMICAS, SOCIALES Y POLÍTICAS DEL PROCESO DE INTEGRAC...TEMA 14.DERIVACIONES ECONÓMICAS, SOCIALES Y POLÍTICAS DEL PROCESO DE INTEGRAC...
TEMA 14.DERIVACIONES ECONÓMICAS, SOCIALES Y POLÍTICAS DEL PROCESO DE INTEGRAC...
 
TALLER DE DEMOCRACIA Y GOBIERNO ESCOLAR-COMPETENCIAS N°3.docx
TALLER DE DEMOCRACIA Y GOBIERNO ESCOLAR-COMPETENCIAS N°3.docxTALLER DE DEMOCRACIA Y GOBIERNO ESCOLAR-COMPETENCIAS N°3.docx
TALLER DE DEMOCRACIA Y GOBIERNO ESCOLAR-COMPETENCIAS N°3.docx
 
Tema 10. Dinámica y funciones de la Atmosfera 2024
Tema 10. Dinámica y funciones de la Atmosfera 2024Tema 10. Dinámica y funciones de la Atmosfera 2024
Tema 10. Dinámica y funciones de la Atmosfera 2024
 
Prueba de evaluación Geografía e Historia Comunidad de Madrid 2º de la ESO
Prueba de evaluación Geografía e Historia Comunidad de Madrid 2º de la ESOPrueba de evaluación Geografía e Historia Comunidad de Madrid 2º de la ESO
Prueba de evaluación Geografía e Historia Comunidad de Madrid 2º de la ESO
 
PLAN DE REFUERZO ESCOLAR MERC 2024-2.docx
PLAN DE REFUERZO ESCOLAR MERC 2024-2.docxPLAN DE REFUERZO ESCOLAR MERC 2024-2.docx
PLAN DE REFUERZO ESCOLAR MERC 2024-2.docx
 
Análisis de los Factores Externos de la Organización.
Análisis de los Factores Externos de la Organización.Análisis de los Factores Externos de la Organización.
Análisis de los Factores Externos de la Organización.
 
Proyecto de aprendizaje dia de la madre MINT.pdf
Proyecto de aprendizaje dia de la madre MINT.pdfProyecto de aprendizaje dia de la madre MINT.pdf
Proyecto de aprendizaje dia de la madre MINT.pdf
 
CONCURSO NACIONAL JOSE MARIA ARGUEDAS.pptx
CONCURSO NACIONAL JOSE MARIA ARGUEDAS.pptxCONCURSO NACIONAL JOSE MARIA ARGUEDAS.pptx
CONCURSO NACIONAL JOSE MARIA ARGUEDAS.pptx
 
SISTEMA RESPIRATORIO PARA NIÑOS PRIMARIA
SISTEMA RESPIRATORIO PARA NIÑOS PRIMARIASISTEMA RESPIRATORIO PARA NIÑOS PRIMARIA
SISTEMA RESPIRATORIO PARA NIÑOS PRIMARIA
 

Ejercicios Microeconomia

  • 1. Marketing y dirección de empresas 14 MICROECONOMIA  Beas Reátegui, Isabel  Figueiredo Aguilar, Gianella  Toledo Quispe, Maricruz
  • 2. EJERCICIOS 1. En el análisis de un intercambio entre dos personas entre dos personas, suponga que ambas tiene la misma preferencia. ¿Será la curva de contrato una línea recta? Explique su respuesta. ¿Puede dar un ejemplo en él no lo sea? 2. Cite un ejemplo de las condiciones en las que la frontera de posibilidades de producción podría no ser cóncava. 3. en monpsonista compra trabajo por un salario inferior al competitivo. ¿Qué tipo de ineficiencia provocará esta utilización del poder nonopsinio? ¿En qué cambiaría su respuesta si el monopolista en el mercado de productos? 4. Juana tiene 8 litros de bebidas refrescantes y 2 sándwiches. Bartolo, en cambio, tiene 2 lito de bebidas refrescantes y 4 sándwiches. Con estas dotaciones, la relación marginal de sustitución (RMS) de Juana de sándwiches por bebidas refrescantes en tres y la de Bartolo es uno. Represente una caja de edgaworth para mostrar si esta asignación de los recursos es eficiente. En caso afirmativo, explique por qué. En caso negativo indique qué intercambio mejoraría el bienestar de las dos partes. 5. La empresa Acme produce x e y unidades de los bienes alfa y beta, respectivamente. a. Utilice una frontera de posibilidades de producción para explicar por qué la disposición a producir una cantidad mayor o menor de alfa depende de la relación marginal de transformación de beta en alfa. b. Considere dos casos extremos de producción: (i) Acme produce inicialmente cero unidades de alfa o (ii) produce inicialmente cero unidades de beta. Si siempre trata de permanecer en su frontera de posibilidades de producción, describa las posiciones iniciales de los casos (i) y (ii). ¿Qué ocurre cuando la empresa Acme empieza a producir ambos bienes? 6. En nuestro análisis de las caja de producción de Edgeworth, suponga que un nuevo invento hace que el proceso de rendimientos constantes d escala s convierte crecientes. ¿Cómo afecta este cambio a la curva de contratos correspondiente a la producción? 7. Suponga que el oro (G) y la lata (S) son mutuamente sustitutivas porque ambos sirven para protegerse de las inflaciones. Suponga también que las ofertas de los dos metales se mantienen fijas a corto plazo (Qg = 50 y Qs = 200) y que las demandas de oro y plata vienen dadas por las siguientes ecuaciones . Pg = 850 - Qg + 0,5Ps Ps = 540 - Qs + 0,2Pg a. ¿Cuáles son los precios de equilibrio del oro y la plata? b. Suponga que un nuevo descubrimiento de oro eleva la cantidad ofrecida en 85 unidades. ¿Cómo afecta este descubrimiento tato al precio del oro como al de la plata? EJERCICIOS
  • 3. 1. Muchos consumidores consideran que las marcas conocidas son una señal de calidad y pagan más por los productos de marca (por ejemplo, la aspirina, la aspirina Bayer en lugar de una aspirina genérica o las verduras congeladas de marca en lugar de las que llevan la marca del supermercado). ¿Puede ser la marca una señal útil de calidad? ¿Por qué sí o por qué no? 2. Gabriel ha acabado sus estudios universitarios recientemente. Después de trabajar seis meses en su nuevo empleo, finalmente ha ahorrado lo suficiente para comprarse su primer automóvil. a. Tiene muy poca información sobre la diferencias entre las marcas y modelos. ¿Cómo puede utilizar las señales de mercado, la reputación o la estandarización para hacer comparaciones? b. Usted trabaja en le departamento de préstamos de un banco. Después de seleccionar un automóvil Gabriel acude a usted en busca de un préstamo. Como se ha licenciado recientemente no tiene un largo historial crediticio. A pesar de eso, l banco lleva mucho tiempo financiando automóviles a personas que se han licenciado recientemente. ¿Es útil esta información en el caso de Gabriel? En caso afirmativo, ¿Cómo? 3. Una importante universidad prohíbe la calificación de suspenso. Defiende su medida alegando que los estudiantes tienden a rendir por encima de la media cuando no están sometidos a presiones de suspenso. La universidad declara que quiere que todos los estudiantes reciban las calificaciones de sobresaliente y notable. Si el objetivo es elevar las calificaciones globales a notable o a un nivel superior, ¿ es buena esta política?. Analice el ejercicio en relación con el problema de riesgo moral. 4. El profesor Jiménez acaba de ser contratado por el departamento de economía de un importe universidad. El rector ha declarado que la universidad de ha comprometido a dar una educación de alta calidad a sus estudiantes universitarios. Transcurridos los dos primeros meses del semestre, el profesor todavía no ha iniciado sus clases. Parece que está dedicándose por completo a las investigaciones económicas y descuidando la enseñanza. Sostiene que su investigación dará más prestigio al departamento y a la universidad. ¿Debería permitirse que continúa dedicándose exclusivamente a la investigación? Analicé este ejercicio en relación con el problema del principal y del agente. 5. Dada su fama de producir automóviles con insatisfactorios historiales de reparaciones han ofrecido amplias garantías a los compradores (por ejemplo, una garantía durante siete años de todas las piezas y la mano de obra relacionadas con problemas mecánicos). a. Dados sus conocimientos sobre el mercado de <<cachorros>>, ¿Por qué es razonable esta política? b. ¿Es probable que plantee un problema de riesgo moral? Explique su respuesta. 6. Para fomentar la competencia y el bienestar de los consumidores, las autoridades prohíben la publicidad engañosa. ¿Cómo se fomenta la competencia con publicidad
  • 4. verás? ¿Por qué sería menos competitivo un mercado si las empresas realizaran publicidad engañosa? 7. Una compañía de seguros está considerando la posibilidad de crear tres tipos de pólizas de seguros contra incendios: (i) cobertura completa, (ii) cobertura completa por encima de los primeros 10.000 dólares de pérdidas y (iii) cobertura del 90 por ciento de todas las pérdidas. ¿Qué póliza tiene más probabilidades de planear problemas de riesgo moral? 8. El lector ha visto que la información asimétrica puede reducir la calidad media de los productos que se venden en el mercado, ya que los productos de mala calidad expulsan a los de buena calidad. En los mercados en los que predominan la información asimétrica, ¿Estaría usted de acuerdo con cada una de las medidas siguientes o discreparía? Explique brevemente su respuesta. a. El Estado debería subvencionar a las organizaciones de consumidores. b. El Estado debería imponer normas de calidad, por ejemplo, debería prohibir a las empresas la venta de artículos de mala calidad. c. El productor de un artículo de buena calidad probablemente querrá ofrecer una garantía amplia. d. El Estado debería obligar a todas las empresas ofrecer amplias garantías. 9. Dos vendedores de automóviles usados compiten uno al lado del otro en una carretera principal. El primero, Automóviles Hernández, vende automóviles de buena calidad que inspecciona minuciosamente y, si es necesario, repara. La compra y la reparación de cada uno de los automóviles que vende le cuesta, en promedio, 5.000 dólares solamente. Si los consumidores conocieran la calidad de los automóviles usados que compran, pagarían encantados 10.000 dólares, en promedio, por los que vende Hernández y sólo 7.000, en promedio, por los que vende López. Desgraciadamente, los vendedores son demasiado nuevos para tener reputación, por lo que los consumidores no conocen la calidad de sus automóviles. Se imaginan, sin embargo que tienen un 50 por ciento de probabilidades de acabar comprando un automóvil de buena calidad, cualquiera que sea el vendedor al que acudan y, por lo tanto, están dispuestos a pagar 8.500 dólares, n promedio, por un automóvil. Hernández tiene una idea: ofrecer una garantía total por todos los automóviles que vende. Sabe que una garantía que dure Y años le costará 500Y dólares en promedio y que si López trata de ofrecer esa misma garantía. ésta le costará 1.000Y dólares, en promedio. a. Supongamos que Hernández ofrece una garantía de un año por todos los automóviles que vende. ¿Será ésta una señal creíble de calidad? ¿Hará o n López la misma oferta de tal manera que los consumidores puedan suponer correctamente que los automóviles de Hernández son de buena calidad debido a la garantía y que, por lo tanto, valen 10.000 dólares en promedio? b. ¿Qué ocurre si Hernández ofrece una garantía de dos años por sus automóviles? ¿Será está una señal creíble de calidad? ¿Y si ofrece una garantía de tres años? c. Si tuviera que aconsejar a Hernández, ¿Qué periodo de garantía le instaría a ofrecer? Explique por qué.
  • 5. 10. El ingreso a corto plazo de una empresa viene dado por I= 10e - e.e, donde e es el nivel de esfuerzo del trabajador representativo (se supone que todos son idénticos). Un trabajador eligeel nivel de esfuerzo que maximiza su salario, una vez descontado el esfuerzo w - e (se supone que el coste unitario del esfuerzo es 1). Halle el nivel de esfuerzo y el nivel de beneficio (el ingreso menos el salario pagado) correspondiente a cada uno de los siguientes sistemas salariales. Explique por qué estas diferentes relaciones del principal y el agente generan distinto resultados. a. w = 2 cuando e >/ 1; de lo contrario, w = 0. b. w = I /2. c. w = I/12.5. EJERCICIOS 1. Algunas empresas se han instalado en la parte oeste de una ciudad después de que la parte este fuera ocupada por viviendas monoparentales. Cada empresa produce el mismo producto y emite como consecuencia humos nocivos que afectan negativamente a los residentes de comunidad. a. ¿Por qué existe una externalidad creada por las empresas? b. ¿Cree usted que la negociación privada puede resolver el problema de la externalidad? Explique su respuesta. c. ¿Cómo podría averiguar la comunidad el nivel eficiente de calidad del aire? 2. Un programador informático presiona en contra de que los programas informáticos tengan derechos de autor. Sostiene que todo el mundo debería beneficiarse de los programas innovadores hechos para computadoras personales y que la exposición a una amplia variedad de programas inspira a los programadores jóvenes y los lleva a crear programas aún más innovadores. Considerando los beneficios sociales marginales que se derivan posiblemente de su propuesta, ¿Está usted de acuerdo con la postura del programador? 3. Suponga que los estudios científicos suministran la siguiente información sobre los beneficios y los costes de las emisiones de dióxido: Beneficios de la reducción de las emisiones: BM = 400 - 10A Costes de la reducción de las emisiones: CM = 100 - 20A Donde A es la cantidad reducida en millones de toneladas y os beneficios y los costes se expresan en dólares por tonelada. a. ¿Cuál es el nivel de reducción de las emisiones socialmente eficiente? b. ¿Cuáles son el beneficio marginal y el coste marginal de reducción correspondiente al nivel de reducción socialmente eficiente?
  • 6. c. ¿Qué ocurre con los beneficios sociales netos (los beneficios menos los costes) si reducimos un millón de toneladas más que el nivel eficiente? ¿Un millón menos? d. ¿Por qué es socialmente eficiente igualar los beneficios marginales y los costes marginales en lugar de reducir la contaminación hasta que los beneficios totales sean iguales a los costes totales? 4. Cuatro empresas situadas en diferentes puntos de un río vierten distintos cantidades de residuos en él. Estos afectan negativamente la calidaddel agua en al que nadan los propietarios de viviendas que viven en sus márgenes. Estas personas pueden construir piscinas para evitar nadar en el río y las empresas pueden construir piscinas parar evitar nadar en el río y las empresas pueden comprar filtros que eliminen las sustancias químicas perjudiciales que contienen los residuos que vierten en el río. Como asesor de un organismo de planificación regional. ¿Cómo compararía y contrataría las siguientes opciones para hacer frente al efecto perjudicial de los vertidos? a. Una tasa uniforme sobre los vertidos de las empresas situadas en el río. b. Una norma uniforme sobre el nivel se residuos que puede verter cada empresa. c. Un sistema de permisos transferibles de vertidos, en el que el nivel agregado de vertidos sea fijo y todas las empresas reciban permisos idénticos. 5. Las investigaciones médicas han mostrado los efectos negativos que produce el tabaco en la salud de los fumadores <<pasivos>>. Las tendencias sociales recientes apuntan a una creciente intolerancia hacia el consumo de tabaco en zonas públicas. Si usted fuma y desea seguir fumando a pesar del endurecimiento de la legislación contra el tabaco, describa el efecto que produciría las siguientes propuestas legislativas en su conducta. Como consecuencia de estos programas, ¿Se beneficia usted como fumador individual? ¿Se beneficia la sociedad en su conjunto? a. Se propone una ley que reduciría los niveles de alquitrán y nicotina de todos los cigarros. b. Se establece un impuesto sobre cada paquete de los cigarrillos. c. Se establece un impuesto sobre cada paquete de los cigarrillos. d. Los fumadores deben llevar siempre su permiso para fumar expendido por el Estado. 6. Un apicultor vive al lado de un manzanar, cuyo dueño de beneficia de las abejas porque cada colmena poliniza alrededor de un acre de manzanos. Sin embargo, el dueño del manzanar no paga nada por este servicio, porque las abejas acuden al manzanar sin que él tenga nada que hacer. Como no hay suficientes abejas para polinizar todo el manzanar, su dueño debe completar la polinización por medios artificial con un coste de 10 dólares por acre de árbol. La apicultura tiene un coste marginal CM = 10 + 2Q, donde Q es el número de colmenas. Cada colmena produce miel por valor de 20 dólares. a. ¿Cuántas colmenas mantendrá el apicultor? b. ¿Es económicamente eficiente este número de colmenas? c. ¿Qué cambios harían que esta actividad fuera más eficiente?
  • 7. 7. Existen tres grupos en una comunidad. Sus curvas de demanda de televisión pública en horas de programación, T, vienen todas por. W1 = 150 $ - T W2 = 200$ - 2T W3 = 250$ - T Supongamos que la televisión pública es un bien público puro que puede producirse con un coste marginal constante de 200 $ dólares por hora. a. ¿Cuál es el número eficiente de horas de televisión pública? b. ¿Cuánta televisión pública suministraría un mercado privado competitivo? 8. Reconsidere el problema de recursos comunes del Ejemplo 18.5. Suponga que la popularidad de los cangrejos de río continúa aumentando y que la curva de demanda se desplaza de C = 0,401 - 0,0064F a C = 0,50 - 0,0064F. ¿Cómo afecta este desplazamiento de la demanda a la captura afectiva de cangrejos, a la captura eficiente y al coste social del acceso común? Pista: utilice las curvas de coste social marginal y de coste privado del ejemplo. 9. El banco Georges, zona pesquera sumamente productiva de la aguas de Nueva Inglaterra, puede dividirse en dos zonas en función de la cantidad de peces. La zona 1 tiene una cantidad mayor por millas cuadrada, pero la pesca muestra grandes rendimientos decreciente. Las capturas diarias (en toneladas) son en la zona 1. F1 = 200(X1) - 2(X1) (X1). donde X1 es el número de barcos pesqueros que faenan en ella. La zona 2 tiene menos peces por millas cuadrada, pero es mayor y los rendimientos decrecientes no son un problema tan serio. Sus capturas diarias son. F2 = 100(X2) - (X2) (X2). donde X2 es el número de barcos pesqueros que faenan en esa zona. La captura marginal de pescado, CMF, de cada zona pude representarse de la forma siguiente: CMF1 = 200 - 4(X1) CMF2 = 100 - 2(X2) Actualmente hay 100 barcos que tienen licencia para faenar en estas dos zonas. El pescado se vende a 100 dólares la tonelada. El coste total (de capital y de explotación) por barco es constante e igual a 1.000 dólares diarios. Responda a las siguientes preguntas en relación con esta situación. a. Si se autoriza a los barcos a pescar donde quieran sin restricción alguna por parte del Estado, ¿cuántos pescarán en cada zona? ¿Cuál será el valor bruto de las capturas? b. Si el gobierno puede restringir el número de barcos, ¿Cuántos debe asignar cada zona? ¿Cuál será el valor de las capturas? Suponga que el número total de barcos sigue siendo el 100. c. Si aumenta el número de pescadores que quieren comprar barcos y sumarse a la flota pesquera, ¿debe concederles licencia un gobierno que desee maximizar el valor neto de las capturas? ¿Por qué sí o por qué no?
  • 8. RESPUESTAS DE ALGUNOS EJERCICIOS CAPÍTULO 1 1. a. Falso. En Estados Unidos. las posibilidades de sustitución de unas regiones geográficas por otras son escasas o nulas. Por ejemplo, un consumidor de los Ángeles no viajará a Houston (Atlanta) o a Nueva York para comer simplemente porque los precios de las hamburguesas sean más bajos en esas ciudades. Asimismo, un McDonald's o un Burger King de Nueva York no puede ofrecer hamburguesas en Los Ángeles. En otras palabras, una subida del precio de los restaurantes de comida rápida de Nueva York no afectará ni la cantidad demandada ni a la cantidad ofrecida de los Ángeles u otras partes del País. d. Falso. Aunque es improbable que los consumidores recorran todo el país para comprar ropa, los oferentes pueden transportar fácilmente la ropa de una parte del país a otra. Por lo tanto. si los precios de la ropa fueran considerablemente más altos en Atlanta que en los Ángeles, las empresas de ropa transportarían la ropa Atlanta, lo que reduciría el precio en esa ciudad. e. Falso. Aunque algunos consumidores sean fieles a la Coca-Cola o a la Pepsi, hay muchos que sustituirían una por otra en función de las diferencias de precios. Por lo tanto. hay un único mercado de bebidas de cola. CAPÍTULO 2 1. En 1998, Qd = 3.244 - 283P y Qs = 1.944 + 207P. Con los nuevos mercados, Qd = Qd + 200 = 3.444 - 283P, Qs =Qs, 3.444 - 283P = 1.944 + 207P, 1.500 = 490P y P* = 3,06 dólares. A P*, Q* = 3.244(3,06) = 2.378,02. Por lo tanto, P = 3,06 dólares, Q = 2.378. 10. a) La demanda total es Q = 3.244 - 283P; la demanda interior es Qd = 1.700 - 107P; restando la demanda de exportación Qe = 1.544 - 176. El precio inicial de equilibrio del mercado (citado en el ejemplo) es P* = 2,65 dólares, Con una disminución de la demanda de exportación de 40 por ciento la demanda toral se convierte en Q =Qd + 0.6QE = 1.700 - 107P + 0.6(1.544 - 176P) = 2.626,4 - 212,6P. La demandad es igual a la oferta. Por lo tanto, 2.626,4 -212,6P = 1.944 + 207P 682,4 = 419,6P Por lo tanto, P = 682,4/419,6 = 1, 626 dólares, o sea, 1,63 dólares. A este precio, Q = 2.281. Sí los agricultores deberían preocuparse. Con esta disminución de la cantidad y del precio, el ingreso pasa de 6.609 millones de dólares a 3.718 millones.
  • 9. f. Si el gobierno de Estados Unidos apoya un precio de 3,50 dólares, el mercado no se encuentra en equilibrio. A este precio, la demanda es igual a 1.700 - 107(3,50) = 1.325,5 y la oferta es 1.944 + 207(3,50) = 2.668,5. Hay un exceso de oferta (1.343) que el gobierno debe comprar, lo que cuesta 3,50$(1.343) = 4.700,5 millones de dólares. 11. a. En primer lugar, considerando la oferta de los países que no pertenecen a la OPEP, Sc = Q* = 13. Si Es= 0,10 y P*= 18, Es = d (P*/Q*) implica que d = 0,07. Sustituyendo d, Sc y P por sus valores en la ecuación de oferta, c = 11,74 + 0,07P. Asimismo dado que Qd =23, Ed = -b (P*/Q*) = -0,05 y b = 0,06. Si sustituimos b por este valor, Qd por 23 y P por 18 en la ecuación de demanda, significa que 23 = a -0,06(18), por lo que a = 24,08. Por lo tanto, Qd = 24,08 - 0,06P. g. Al igual que antes, Es = 0,4 y Ed = -0,4: Es = d (P*/Q*) y Ed = -b (P*/Q*), lo que implica que 0,4 = d(18/13) y -0,4 = -b(18/23). por lo tanto, d = 0,29 y b = 0,51. A continuación despejamos c y a: Sc = c + dPy Qd = a - bP, lo que implica que 13 = c + (0,29)(18) y 23 = a - (0,51)(18). Por lo tanto, c = 7,78 y a = 32,18. CAPÍTULO 3 a. Véase la figura 3(a), en la que B representa el número de paquetes de mantequilla y M el de margarina. b. La convexidad significa que la curva está <<combada hacia adentro>>. En este caso las curvas de indiferencia no son <<estrictamente convexas>>, ya que son líneas rectas.
  • 10. Figura 3(a) c) La restricción presupuestaria es Y= P BB + P M, 20 = 2B + MM, B = 10 – 0,5M. Dado que Guille se muestra indiferente entre la mantequilla y la margarina y que el precio de la mantequilla es más alto que el de la margarina, sólo comprará margarina. 6. a) Véase a la figura 3(b), en la que A representa la cantidad de bebidas alcohólicas y N representa la cantidad de bebidas no alcohólicas. b) Cualquiera que sea la combinación de A y N, Juárez está dispuesto a renunciar a una cantidad menor de A para conseguir alguna de N en comparación con Sanz. Por lo tanto, la RMS entre A y N de Juárez es menor que la de Sanz. Las curvas de indiferencia de Juárez son menos inclinadas que las de Sanz en cualquier punto del gráfico. c) Para maximizar la satisfacción, cada consumidor debe consumir unas cantidades tales que la RMS entre dos mercancías cualquiera sea igual a su relación de precios. Sus RMS deben ser iguales porque se enfrentan a los mismos precios. Pero como tienen preferencias distintas, consumirán cantidades diferentes de los dos bienes A y B. 8. En la figura 3(c), representamos en los ejes las millas que vuela, M, y todos los demás bienes, B, expresados en dólares. La pendiente de la recta presupuestaria es –PM/PB. El precio de las millas recorridas varia cuando varían estas, por lo que la restricción presupuestaria tiene un vértice en 25.000 y 50.000 millas. Supongamos que PM es 1 dólar por milla cuando <25.000 millas, PM = 0,75 dólares cuando 25.000< M< 50.000 y PM = 0,50 dólares cuando M>50.000. Supongamos también que PB =1 dólar. En ese caso, la pendiente del primer segmento es -1, la del segundo es -0,75 y la del último es -0,5.
  • 11. CAPITULO 4 1. a) Cuando el precio experimenta una pequeña variación, es apropiada la siguiente formula de la elasticidad- punto: EF = % Q/% P. En el caso de los chips de computadora, EP= -2, y en el de las unidades de disco, EP= -1. Sea IT= PQ el ingreso antes de la variación de los precios e (IT= PQ)2 el ingreso después de las variación de los precios. En el caso, IT=IT2 – IT. En el caso de los chips de computadora, IT= -12%IT. En el caso de las unidades de disco, IT= -1%IT. b) Aunque conocemos la sensibilidad de la demanda a las variaciones del precio, necesitamos conocer las cantidades y los precios de los productos para hallar los ingresos totales derivados de las ventas. 5. a) La curva de demanda es una línea recta que tiene una ordenada en el origen de P=12 y una abscisa en el origen de Q=6 (ya que Q =6 – P/2). b) Si no hubiera peaje, el precio P seria 0, por lo que Q=6. c) Si el peaje es de 6 dólares, Q=3. La pérdida de excedente del consumidor es de la diferencia entre el excedente del consumidor correspondiente a P=0(36) y el excedente del consumidor correspondiente a P=3(9), o sea, 27.
  • 12. 10. Si las variaciones del precio son pequeñas, seria adecuada la fórmula de la elasticidad-punto. Pero en este caso el precio de los alimentos se duplica en 2 dólares a 4, por lo que debe utilizarse la elasticidad-arco: Ep= ( Q/ P) P/Q). Sabemos que EP= -1, P=2, P=2 y Q=5.000. Por lo tanto, si la renta no varía, podemos hallar Q: -1 = ( Q/2)(2+1)/(5.000+ Q/2)= ( _Q.3)(10.000+ Q). Observamos que Q=- 2.500: reduce su número de consumo de alimentos de 5.000 a 2.500 unidades. a) Una devolución de impuestos de 5.000 dólares implica un aumento de la renta de5.000 dólares. Para calcular la respuesta de la demanda a la devolución de impuestos, utilizamos la definición de la elasticidad-arco con respecto a la renta: E1 = 0,5, I= 25.000, I= 5.000, Q= 2.500. Hallamos Q: 0,5= ( Q/5.000)(25.000+2.500)/(2.500+ Q/2). Dado que Q=238, aumenta su consumo de alimentos de 2.500 unidades a 2.738. b) En su curva final de indiferencia, decide consumir 2.738 unidades de alimentos (lo que le supone un gasto de 10.952 dólares) y una cantidad de todos los demás bienes por valor de 19.048 dólares. Al precio inicial de los alimentos de 2 dólares, esta combinación le habrá costado 2.738.2$+19.048$= 24.524 dólares. Por lo tanto, habría tenido 476 dólares más para gastar en alimentos o en otros bienes de consumo y habría disfrutado de un bienestar mayor. CAPITULO 4 – APENDICE 1. La primera función de utilidad puede representarse como una serie de líneas rectas; la segunda como una serie de hipérbolas en el cuadrante positivo; y la tercera como una serie de curvas en forma de L. La única función de utilidad que es estrictamente convexa es la segunda. 3. La ecuación de Slutsky es dX/dPx =aX/aP*/ u=u* - X ( X/ I), donde el primer término representa el efecto-sustituto y el segundo el efecto-renta. Como no existe sustitución cuando varia el precio con ese tipo de función de utilidad, el efecto sustitución es cero. CAPITULO 5 2. El cuadro 5 adjunto indica los 4 estados mutuamente excluyentes 7. Los consumidores que tengan la renta X* compraran simultáneamente un seguro injusto y harán apuestas injustas. Son reacios a experimentar grandes pérdidas de renta, pero amantas de obtener grandes ganancias.
  • 13. El congreso aprueba el arancel El congreso no aprueba el arancel Baja tasa de crecimiento Estado 1: Estado 2: Lento crecimiento con arancel Lento crecimiento sin arancel Elevada tasa de crecimiento Estado 3: Estado 4: Rápido crecimiento con arancel Rápido crecimiento sin arancel CAPITULO 6 1. a) El producto medio del trabajo, PM, es igual a Q/L. El producto marginal del trabajo, PM, es igual a: Q/ L. El cuadro adjunto indica los cálculos relevantes. b) En este proceso de producción, el trabajo muestra rendimientos decrecientes, característicos de todas las funciones de producción en las que hay un factor fijo. Cada unidad adicional de trabajo genera un aumento menor de la producción que la anterior. c) El producto marginal del trabajo puede ser negativo cuando hay congestión en la fábrica de sillas. A medida que aumenta el número de trabajadores que utilizan una cantidad fija de capital, se molestan, por lo que disminuye la producción. 5. Si el producto marginal del trabajo (PM) es mayor que el producto (PMe), cada unidad adicional de trabajo es más productiva que la media de todas las unidades anteriores. Añadiendo la última unidad, aumenta la medida de todas ellas. El PMe es máximo cuando la producción de la última es igual a la medida de todas las anteriores.
  • 14. 8. a) Sea Q la producción de DISK, Inc., Q la producción de FLOPPY, Inc., y X las cantidades idénticas de capital de trabajo de las dos empresas. En ese caso Q=10X X= 10X y Q=10X X = 10X = 10X. Como Q = Q, las dos empresas generan la misma producción con los mismos factores. b) Con una cantidad fija de capital de 9 máquinas, las funciones de producción se convierten en Q=30L y Q = 37,37L. Considere el cuadro adjunto: El producto marginal del trabajo en cada unidad de trabajo superior a 1 unidad es mayor en el caso de DISK, Inc. CAPITULO 7 3. a) El coste total, CT, es igual al coste fijo, CF, más el coste variable, CV. Como la franquicia, F, es una cantidad fija, los costes fijos de la empresa aumentan en la cuantía de la misma. En ese caso, el coste medio, igual a (CF +CV)/Q, y el coste fijo medio igual a (CF/Q), aumentan en la cuantía de la franquicia media (F/Q). El coste variable medio no resulta afectado por la franquicia, como el coste marginal. b) Cuando se establece un impuesto t, los costes variables aumentan es tQ. El coste variable medio aumenta en t (el coste fijo es constante), al igual que el coste (total) medio. Como el coste total aumenta en t con cada unidad adicional, el coste marginal aumenta en t. 4. Probablemente se refiere al beneficio contable; se trata del concepto que se utiliza habitualmente en la mayoría de los análisis sobre los resultados financieros de la empresa. En ese caso, el artículo indica que existe una diferencia considerable entre los beneficios contables y los económicos. Sostiene que según el convenio colectivo en vigor, los fabricantes de automóviles deben pagar a muchos trabajadores aun cuando no trabajen, lo cual implica que sus salarios son IRECUPERABLES durante la vigencia del convenio. Los beneficios contables se calcularían restando los salarios pagados, pero los beneficios económicos no, puesto que hemos dicho que son costes irrecuperables. Por lo tanto, es
  • 15. posible que los fabricantes de automóviles obtengan beneficios económicos por sus ventas, aun cuan do experimentan pérdidas contables. 5. Si la empresa puede producir una silla con 4 horas de trabajo o 4 de maquinaria o cualquier combinación, la isocuenta es una línea recta cuya pendiente es -1 y los puntos de intersección con los ejes son K = 4 y L =4. La recta isocoste, CT = 22L + 110K, tiene una pendiente de 1/5 y unos puntos de intersección con los ejes de K = CT/110 y L = CT/22. El punto minimizado del coste es una solución de esquina, donde L = 4 y K = 0 y CT = 88. 8. La producción de gasolina exige destilar el crudo y refinarlo para convertirlo en gasolina. Dado que el coste marginal de producción es constante hasta que se alcanza el límite de la capacidad en el caso de ambos procesos, las curvas de coste marginal tienen forma de L vista en el espejo. El coste marginal total, CM=CM+CM, donde CM, es el coste marginal de destilar crudo hasta alcanzar el límite de la capacidad, Q y CM es el coste marginal de refinar el crudo destiladohasta alcanzar el límite de la capacidad, Q. Si la capacidad máxima de la unidad de destilación es menor que la de la unidad de pirolisis, el CM total es vertical en Q (véase la figura 7). Si la capacidad máxima de la unidad de pirolisis es menor que la de unidad de destilación, el CM total es vertical en Q. CAPITULO 7 - APENDICE 1. a. El concepto de rendimientos de escala se refiere a la relación entre la producción y los aumentos proporcionales de todos los factores. Si F (L, K) F (L, K), hay rendimientos crecientes de escala; si F (L, K) =F (L, K), hay rendimientos constantes de escala; si F (L, K) F
  • 16. (L, K), hay rendimientos decrecientes de escala. Aplicando esta definición a F (L, K) = KL, F (L, K) = (K) (L) = KL = F (L, K) F (L, K). Por lo tanto, esta función de producción muestra rendimientos crecientes de escala. b. F (L, K) = 10K + 5L = F (L, K). la función de producción muestra rendimientos constantes de escalas. c. F (L, K) = (KL) = () = (KL) = (KL) = F (L, K). La función de producción muestra rendimientos constates de escala. 2. El producto marginal del trabajo es 100K. El producto marginal del capital es 100L. La relación marginal de sustitución técnica es K/L. Igualemos esta relación al cociente entre el salario y la tasa de alquiler del capital: K/L = 30/120 o sea L = 4K. A continuación sustituimos L en la función de producción por su valor y despejamos el valor de K con el que se obtiene un nivel de producción de 1.000 unidades = 100K 4K. Por lo tanto, K = 2,5 y el coste total es igual a 379,20 dólares. CAPITULO 8 1. El cuadro adjunto muestra la información sobre el ingreso y los costes de la empresa cuando el precio baja a 35 dólares. A un precio de 35 dólares, la empresa debería producir 7 unidades para maximizar los beneficios 3. a. Los beneficios se maximizan cuando el coste marginal (CM) es igual al ingreso marginal (IM). En este caso, IM es igual a 60 dólares. Igualando CM a 60 se obtiene una cantidad máximizadora de los beneficios de 30. b. Los beneficios son iguales al ingreso total (PQ) menos el coste total. Por lo tanto, PQ – 100 – Q. Si P = 60 y Q = 30, beneficios = 800. c. La empresa produce a corto plazo si sus ingresos son mayores que sus costes variables. La curva de oferta a corto plazo de la empresa es el tramo de su curva CM situado por encima del CVMe mínimo. En este caso, CVMe es el coste variable, Q2, dividido por la cantidad, Q. Por lo tanto, CVMe = Q. Además, CM es igual a 2Q. Por lo tanto, CM es mayor que CVMe cuando las cantidades son superiores a 0. Eso significa que la empresa produce a corto plazo mientras el precio sea positivo. 5. a. Cuando se establece un impuesto de 1 dólar sobre una única empresa, todas sus curvas de costes se desplazan en sentido ascendente en 1 dólar. b. Como la empresa es precio-aceptante, el establecimiento de un impuesto sobre una empresa solamente no altera el precio de mercado. Dado que la curva de oferta a corto plazo de la empresa es su curva de coste marginal (por encima del coste variable medio) y que la curva de coste marginal se ha desplazado en sentido ascendente (o hacia adentro), la empresa ofrece menos al mercado a todos los precios. c. Si se establece un impuesto sobre una única empresa, esta quebrara a menos que obtuviera unos beneficios económicos positivos antes de que se estableciera el impuesto.
  • 17. CAPITULO 9 1. a. En el equilibrio de libre mercado, L = L. Por lo tanto, w = 4 dólares y L = L = 40. Si el salario mínimo es de 5 dólares, L = 50 y L = 30. El número de personas empleadas viene dado por la demanda del trabajo. Por lo tanto, los empresarios contratan 30 millones de trabajadores. b. Con una subvención, la empresa solo paga w-1. La demanda de trabajo se convierte en L = 80 – 10(w – 1). Por lo tanto, w = 4,5’ dólares y L = 45. 4. a. Igualando la demanda y la oferta, 28 – 2P = 4 + 4P. P = 4 y Q = 20. b. La reducción del 25 por ciento exigida por el nuevo programa de pago en especie implicaría que los agricultores producen 15.000 millones de búshels. Para animarlos a reducir la superficie cultivada, el gobierno debedarles 5.000 millones de búshels que venden en el mercado. Como la oferta total del mercado sigue siendo de 20.000 millones de búshels, el precio del mercado sigue siendo de 4 dólares por búshels. Los agricultores ganan 20.000 millones de dólares (4 dólares multiplicados por 5.000 millones de búshels) con el programa, mientras que los consumidores no resultan afectados. c. Los contribuyentes sales ganando por que el gobierno no tiene que pagar para almacenar el trigo durante un año y transportarlo a un país subdesarrollado. El programa de pago en especie puede durar mientras duren las reservas de trigo. Pero supone que la tierra que deja de cultivarse puede volver a cultivarse cuando se agoten las existencias. En caso contrario, los consumidores podrán acabar pagando más por los productos derivados del trigo. Por último los agricultores obtienen unos beneficios extraordinarios por que no tienen costes de producción. 9. Las curvas de oferta y demanda de gas natural pueden obtenerse aproximadamente de la manera siguiente: Q = 14 + 2P + 0,25P, Q = -5P + 3,75P. Si el precio del petróleo es de 12 dólares el barril, estas curvas se convierten en Q = 17 + 2P y Q = 45 – 5P. Igualando Q y Q, 17 + 2P = 45 – 5P P = 4 dólares. A este precio, la cantidad de equilibrioes de 25 bpc. Si se establece un precio máximo de 1 dólar, los productos ofrecerán 19 bpc y los consumidores
  • 18. demandaran 40. Los consumidores demandan el área A – B = 57 – 3,6 = 53,4 miles de millones de dólares en la figura. Los productores pierden A – C = -57-9=66 miles de millones de dólares. La pérdida irrecuperablede eficiencia es igual a 53,4-66=12,6 miles de millones de dólares. 13. No. El caso más claro es aquel en la que los mercados de trabajo son competitivos. Con cualquiera de los dos tipos de impuestos, la diferencia entre la oferta y la demanda debe ser igual en total al 12,4 por ciento del salario de pagado. Da lo mismo que el impuesto lo paguen totalmente los trabajadores (lo que reemplazaría la curva de oferta efectiva un 12,4 por ciento) o los empresarios (lo que desplazaría la curva de demanda efectiva un 12,4 por ciento). Lo mismo ocurre con cualquier combinación de los dos impuestos que sume 12,4 por ciento. CAPITULO 10 2. Hay tres importantes factores (1) ¿En qué medida son similares los productos que ofrecen los competidores de Caterpillar? Si son sustitutivos cercanos, una pequeña subida de precio podría llevar a los clientes a pasarse a la competencia. (2) ¿Qué antigüedad tiene el parque existente de tractores? Una subida del precio de un cinco por ciento provoca un descenso menor de la demanda si el parque de tractores es más antiguo. (3) En su calidad de capital que intervi esperada del e como factor en la producción agrícola, ¿Cuál es la rentabilidad esperada del sector agrícola? Si la renta agrícola esperada están ion disminuyendo, una subida de los precios de los tractores provoca un descenso mayor de la demanda que el que se estimaría con la información sobre las ventas y los precios pasados. 4. a. La producción óptima se halla igualando el ingreso marginal y el coste marginal. Si la función de demanda es lineal, P=a-bQ (aquí, a=100 y b=0,01), IM=a-2bQ=100-2(0,01) Q. Coste total =30,000+50Q, por lo que Q=2.500. Introduciendo este resultado en la función de demanda, P=100-0,01. 2.500=75 centavos. Los beneficios totales son -30.000+50 2.500- 0,01(2.500)= -30.00+125.000-62.500, o sea, 325 dólares a la semana. b. Suponga que inicialmente los consumidores deben pagar el impuesto. Como el precio (incluido el impuesto) que estarían dispuestos a pagar no varía, la función de demanda puede expresarse de la forma siguiente: P+t=100-0,01Q-t. Como el impuesto eleva el precio de cada unidad, el ingreso total del monopolista disminuye en tQ y el ingreso marginal disminuye en t: IM=100-0,02Q-t, donde t= 10 centavos. Para hallar el nivel de producción maximizador de los beneficios con el impuesto, igualamos el ingreso marginal y el coste marginal: 100-0,02Q-10=5, o sea Q=2.000 unidades. A partir de la función de demanda, 100-0,01 2.000-10=70 (2.000)-(30.000+50(2.000))=10.000 centavos, o sea, 100 dólares a la semana. 8. a. A favor: Aunque Alcoa controlaba alrededor de un 90 por ciento de la producción de aluminio primario de Estados Unidos, la producción de aluminio secundario de la empresas dedicadas al recicladorepresentaba un 30 por ciento de la oferta total de aluminio. Debería ser posible que una proporción mucho mayor de la oferta de aluminio procediera de fuentes secundarias. Por lo tanto, la elasticidad-preciode la demanda de aluminio primario de Alcoa
  • 19. es mucho mayor de lo esperado. En muchas aplicaciones, otros metales, como el cobre o el acero, son sustitutos viables del aluminio. En este caso, la elasticidad de la demanda a la que se enfrenta Alcoa podría ser menor de lo esperado. b. En Contra: La cantidad de oferta potencial es limitada. Por lo tanto, manteniendo un precio alto y estable Alcoa podría obtener beneficios monopolísticos. Por otra parte, como Alcoa producía inicialmente el metal que reaparecía reciclado, habría tenido en cuenta en las decisiones de producción la influencia de la recogida en chatarra en los futuros precios. Por lo Tanto, ejercía un control monopolístico efectivo en la oferta de metal secundario. c. No se pidió a Alcoa que vendiera ninguna de sus instalaciones productivas de Estados Unidos, sino que (1) se le prohibió que pujara por las dos plantas de aluminio primario construidas por el Estado durante la Segunda Guerra Mundial, y (2) se le obligo a deshacerse de su filial canadiense , que se convirtió en Alcoa. 11. No. En un mercado competitivo, una empresa considera que el precio es horizontal e igual al ingreso medio, que es igual que el ingreso marginal. Si el costo marginal de la planta b aumenta, el precio seguirá siendo igual al costo marginal, al coste marginal total y al ingreso marginal de la A. Solo se produce la cantidad de la planta B (lo cual reduce, a su vez, la cantidad total), como muestra la figura 10. CAPITULO 11 1. a. Obligando a los pasajeros a pasar, al menos, el sábado por la noche fuera de casa se separa a los que viajan por motivos de negocios, que prefieren estar de vuelta el fin de semana, de los turistas, que viajan el fin de semana. b. Basando los precios en el lugar de residencia del comprador, se hace la selección geográficamente. En este caso, los precios pueden reflejar los costes de transporte, que el cliente paga independientemente de que el cemento se entregue en el lugar de residencia del comprador o en la cementera. c. Vendiendo procesadores de alimentos juntos con los vales de reembolso se divide a los consumidores en dos grupos: (1) los consumidores que son menos sensibles al precio (aquellos cuya demanda tiene una elasticidad menos) no piden el reembolso; y (2) los clientes que son más sensibles al precio (aquellos cuya demanda tiene una elasticidad más alta) solicitan el reembolso. d. Una reducción temporal del precio del papel higiénico es un tipo de discriminación intertemporal de precios. Los clientes sensibles al precio compran más papel durante el periodo en que se reduce el precio, mientras que los clientes que no son más sensibles al precio compran la misma cantidad. e. El cirujano puede distinguir a los pacientes de renta alta de los pacientes de renta baja negociando. El arbitraje no es un problema porque la cirugía plástica no puede transferirse de los pacientes de renta baja a los de renta alta.
  • 20. 8. a. Un monopolista que tiene dos mercados debe elegir unas cantidades tales en cada uno que los ingresos marginales sean idénticos en los dos e iguales al coste marginal. El coste marginal es la pendiente de la curva de coste total, 30. Para hallar los ingresos marginales de cada mercado, despejaremos el precio en función de la cantidad. A continuación introducimos esta expresión del precio en la ecuación del ingreso total. PNY = 150–3QNY y PLA = 120 – (3/2) QLA. Los ingresos totales son, pues, ITNY = QNYPNY = QNY (150-3QNY) e ITLA = QLAPLA = QLA (120-(3/2)QLA). Los ingresos marginales son las pendientes de las curvas de ingreso total: IMNY = 150-6QNY e IMLA = 120-3QLA. A continuación, igualamos cada ingreso marginal con el coste marginal (=30), lo que implica que QNY = 20 y QLA = 30. Con estas cantidades, hallamos el precio de cada mercado: PNY = 150-3.20=90 y PLA = 120-(3/2).30 = 75. b. Con el nuevo satélite, Sal ya no puede separar los dos mercados. La función total de demanda es la suma horizontal de los dos mercados. Si el precio es superior a 120, la demanda total es simplemente la función de demanda de Nueva York. Si es inferior, sumamos las dos demandas: QT= 50-(1/3) P + 80 – (2/3) P =130 –P. Sal maximiza los beneficios eligiendo una cantidad total tal que IM=CM. El ingreso total es QP, donde P=130- Q.IT = Q (130-Q), por lo que el ingreso marginal es 130-2Q. Igualando dicho ingreso marginal y el coste marginal, se obtiene una cantidad maximizadora de los beneficios de 50 y un precio de 80. En el mercado de Nueva York, la cantidad es igual a 50-(1/3)80=231/3 y en el de Los Ángeles, es igual a 80-(2/3)80 = 262/3 . En total, se compran 50 unidades a un precio de 80. c. En las condiciones de mercado de (a), los beneficios son iguales a la suma de los ingresos de cada mercado menos el coste de producir la cantidad para ambos: QNYPNY + QLAPLA– 1.000-30(QNY+QLA) = 20.90+30.75-1.000+30(20+30) = 1.550. En las condiciones de mercado de (b) los beneficios son iguales al ingreso total menos el coste de producir la cantidad para ambos mercados: QP-(1.000+30Q)=50.80-(1.000+30.50)=1.500. Por lo tanto, Sal gana más dinero cuando se separan los dos mercados. En las condiciones de mercado de (a), en el mercado de Nueva York el excedente del consumidor es (150-90).20 (1/2)=600 y en el de Los Ángeles es (120-75).30 (1/2)=675. En las condiciones de mercados de (b), en el mercado de Nueva York el excedente del consumidor es (150-80)231/3 (1/2)=817 y en el de Los Ángeles es (120-80).262/3 (1/2)=533. Los neoyorquinos prefieren (b) porque el precio de equilibrio es 80 en lugar de 90, por lo que su excedente del consumidor es mayor. Pero los clientes de Los Ángeles prefieren (a) porque el precio de equilibrio es 75 en lugar de 80. 10. a. Si las demandas individuales son Q1 = 6-P, el excedente del consumidor individual es igual a 18 dólares a la semana, o sea, 936 dólares al año. Una cuota de afiliación de 936 dólares captura todo el excedente del consumidor, incluso aunque no se cobre una tarifa por
  • 21. utilizar las pistas, ya que el coste marginal es cero. Los beneficios semanales serian iguales al número de tenistas serios, 1.000, multiplicadopor la cuota semanal de afiliación, 18 dólares, menos 5.000 dólares, que son los costes fijos, o sea, 13.000 dólares semanales. b. Cuando hay dos clases de clientes, el dueño del club maximiza los beneficios cobrando una tarifa por utilizar las pistas superiores al coste marginal y cobrando una cuota de afiliación igual al excedente restante del consumidor que tenga la demanda más baja: el tenista esporádico. La cuota de afiliación, T, es igual al excedente del consumidor restante una vez calculada la tarifa que ha de cobrarse por utilizar las pistas: T= (Q2 – 0) (6-P) (1/2), donde Q2=3-(1/2) P, o sea, T= {3-(1/2) P} (6-P) (1/2)=9-3P + P2 /4. Las cuotas de afiliación totales cobradas a todos los jugadores serian 2.000(9-3P + P2 /4). Los ingresos generados por las cuotas de afiliación son iguales a P (Q1+Q2)= P{1.000(6-P)+1.000(3-P/2)}=9.000P-1.500P2 . Entonces, IT=2.000{(9-3P + P2 /4) + 9.000P-1.500P2 =18.000+3.000P-1.000P2 . El coste marginal es cero y el ingreso marginal viene dado por la pendiente de la curva de ingreso total: IT/ P =3.000-2.000P. Igualando el ingreso marginal y el coste marginal, se obtiene un precio de 1,50 dólares por hora. El ingreso total es igual a 20.250 dólares. El coste total es igual a los costes fijos de 5.000 dólares. Por lo tanto, los beneficios son iguales a 15.250 dólares a la semana, cantidad superior a los 13.000 dólares semanales que se obtienen cuando solo se hacen socios los tenistas profesionales. c. Una cuota de afiliación de 18 dólares semanales solo atraería a los tenistas serios, los ingresos totales serian 54.000 dólares y los beneficios de 49.000 dólares a la semana. Si hay tenistas serios como esporádicos, las cuotas de afiliación serian iguales a 4.000 multiplicado por el excedente del consumidor del tenista esporádico: T=4.000(9-3P+ P2 /4). Las tarifas de la utilización de las pistas son P{(6-P)3.000+(3-P/2)1.000}=(21P-3,5P2 )1.000. Entonces IT {4(9- 3P + P2 /4)+ (21P-3,5P2 )} 1.000= (36+9P-2,5P2 )1.000. Igualando el ingreso marginal y el coste marginal, el precio es de (9/5), o sea, de 1,80 dólares por hora. En este caso, el ingreso total es igual a 44.100 dólares. El coste total es igual a los costes fijos de 5.000 dólares. Los beneficios con una tarifa de dos tramos son de 39.100 dólares semanales, cantidad que es inferior a los 49.000 que se obtienen con tenistas profesionales solamente. El dueño del club debería fijar unas cuotas anuales de 936 dólares y obtener unos beneficios de 2.548 millones de dólares al año. 11. La venta conjunta mixta suele ser la estrategia ideal cuando las demandas solo están correlacionadas algo negativamente o cuando los costes marginales de producción son significativos. Los cuadros adjuntos presentan los precios de reserva de los tres consumidores y los beneficios generados por las tres estrategias. Precio de reservas (dólares) Para el bien 1 Para el bien 2 Total Consumidor A 3,25$ 6,00$ 9,25$ Consumidor B 8,25 3,25 11,50
  • 22. Consumidor C 10,00 10,00 20,00 Precio 1 Precio 2 Venta conjunta Beneficio Venta por separado 8,25 $ 6,00$ ____ 28,50$ Venta conjunta pura ____ ____ 9,25$ 27,75 Venta conjunta mixta 10,00 6,00 11,50 29,00 La estrategia que maximiza los beneficios consiste en vender cada artículo por separado. 15. a. Los precios y los beneficios óptimos correspondientes a cada estrategia son: Precio 1 Precio 2 Venta conjunta Beneficio Venta por separado 40,00 $ 40,00$ ____ 240,00$ Venta conjunta pura ____ ____ 100,00$ 400,00 Venta conjunta mixta 59,95 59,95 100,00 319,90 La venta conjunta pura es superior a la mixta porque cuando los costes marginales son cero, no hay razón alguna para impedir a ningún cliente que compre los dos bienes. b. Con un coste marginal de 35 dólares, los precios y los beneficios óptimos son: Precio 1 Precio 2 Venta conjunta Beneficio Venta por separado 90,00 $ 90,00$ ____ 110,00$ Venta conjunta pura ____ ____ 100,00$ 120,00$ Venta conjunta mixta 59,95 59,95 100,00 110,00$ La venta conjunta mixta sigue siendo superior a todas las demás estrategias. CAPITULO 11 – APENDICE Examinamos cada caso y comparamos los beneficios. a. Las cantidades y los precios óptimosg sin mercado exterior de motores son QMot = QMon = 2.000, PMot = 8.000 dólares y PMon = 18.000 dólares. En el caso de la
  • 23. división de motores, IT= 2.000. 8.000 $ = 16 millones de dólares; CT = 2(2.000) + 16 millones=31 millones de dólares y π = 4 millones de dólares. Los beneficios totales son iguales a 12 millones de dólares. b. Las cantidades y los precios optimos con un mercado exterior de motores son QMot=1.500, QMon=3.000, PMot=6.000 dólares y PMon=17.000 dólares. En el caso de la división de motores, IT=1.500. 6.000 $=9 millones de dólares; CT=2(1.500)2 =4,5 millones de dólares y π=4,5 millones de dólares. En el caso de división de montanje, IT=3.000. 17.000 $=51 millones de dólares; CT=(8.000 + 6.000)3.000=42 millones de dólares y π= 9 millones de dólares. Los beneficios totales son iguales a 13,5 millones de dólares. c. Las cantidades y los precios optimos con un mercado monopolizado de motores son QMot=2.200, QMon=1.600, PMot =8.800 dólares y PMon =18.400 dólares; se venden 600 motores en el mercado monopolizado por 9.400 dólares. En el caso de la división de motores, IT=1.600. 8.800$ + 600. 9.400=19,72 millones de dólares; CT=2(2.200)2 =9,68 millones de dólares y π= 10,04 millones de dólares. En el caso de la división de montaje, IT=1.600. 18.400 $=29,44 millones de dólares; CT=(8.000+8.800)1.600=26,88 millones de dólares y π=2,56 millones de dólares. Los beneficios totales son iguales a 12,6 millones de dólares. La división superior, que construye motores, obtiene unos beneficios máximos cuando tiene el monopolio de los motores. La división inferior, que construye automóviles, obtiene unos beneficios máximos cuando hay un mercado competitivo de motores. Dado el elevado coste de los motores, la empresa obtiene mejores resultados cuando los motores se producen con el menos coste posible con un mercado competitivo exterior de motores. CAPITULO 12 1. Cada empresa obtiene beneficios económicos distinguiendo su marca del resto. Si estos competidores se fusionaran para formar una única empresa, el monopolista resultante no producirá tantas marcas como antes de la fusión. Pero producir varias marcas con precios y características distintas es una manera de dividir el mercado en grupos de clientes con elasticidades-precio diferentes. 2. a. Para maximizar los beneficios π=53Q – Q2 – 5Q, hallamos π/ Q = - 2Q+48=0. Q=24, por lo que P=29. Los beneficios son iguales a 576. b. P=53-Q1-Q2π1=PQ1-C(Q1)=53Q1-Q1 2 -Q1Q2-5Q1 y π=PQ2-C(Q2)=53Q2-Q1Q2- Q2 2-5Q2. c. El problema al que se enfrenta la empresa 1 consiste en maximizar los beneficios, dado que la produccion de la 2 no variara en respuesta a la decision de produccion de la 1. Por lo tanto, la 1 elige el valor de Q1 que maximice π1, al igual que antes. La variación de π1 con respecto a una
  • 24. variación de Q1 es 53-2Q1–Q2-5=0, lo que implica que Q1=24- Q2/2. Dado que el problema es simétrico, la función de reacción de la empresa 2 es Q2=24- Q1/2. d. Hallamos los valores de Q1 y Q2 que satisfacen ambas funciones de reaccion: Q1=24-(1/2)(24-Q1/2). Por lo tanto, Q1=16 y Q2=16. El precio es P=53-Q1-Q2=21. Los beneficios son π2=P.Q1-C (Qi)=256. Los beneficios totales de la industria son π1+π2=512. 5. Verdadero. La curva de reacción de la empresa 2 es q2=7,5-1/2q1 y la curva de reacción de la 1 es q1=15-1/2q2. Sustituyendo, tenemos que q2=0 y q1=15. El precio es 15, que es el precio monopolístico. 11. a. Para hablar el equilibrio de Nash, calculamos la función de reacción de cada empresa y hallamos simultáneamente el precio. Suponiendo que el coste marginal es cero, los beneficios de la empresa 1 son P1Q1= P1 (20- P1+ P2 )=20 P1 – P2 1+P2P1.IM1=20-2P1+P2. Al precio maximizadode los beneficios, IM1=0. Por lo tanto, P1 =(20 + P2)/2. Como las empresas 1 y 2 son simétricas, el precio maximizado de los beneficios de la 2 es P2=(20+P1)/2. Introducimos la función de reacción de la empresa 2 en la de la 1: P1={20+(20+P1)/2}/2=15 + P1/4. P1=20. Por simetría, P2=20. En ese caso, Q1=20 y por simetría Q2=20. Los beneficios de la empresa 1 son P1Q1=400 y los de la 2 también es 400. b. Si la empresa 1 es la primera en fijar su precio, tiene en cuenta la función de reacción de la empresa 2. Los beneficios de la empresa 1 son π1=P1 {20- P1 +(20 + P1)/2}. En ese caso, dπ1/dP1=20-2P1+10+ P1. Igualando esta expresión a cero, P1 =30. Sustituyendo P1 por su valor en la función de reacción de la empresa 2, P2 =25. A estos precios, Q1=20-30+25=15 y Q2=20+30-25=25. Los beneficios son π1=30.15=450 y π2=25.25=625. c. Su primera opción debería ser la (iii) y la segunda debería ser la (ii). Fijar unos precios superiores a los valores del equilibrio de Cournot es opcional para ambas empresas cuando se siguen estrategias de Stackelberg. A partir de las funciones de reacción, sabemos que el líder de precios induce al seguidor al subir el precio. Pero el seguidor sube el precio menos que el líder y, por lo tanto, cobra un precio más bajo que este. Las dos empresas disfrutan de mayores beneficios, pero la seguidora obtiene mejores resultados y las dos obtienen mejores resultados que en el equilibrio de Cournot. CAPITULO 13 1. Si los juegos se repiten indefinidamente y todos los jugadores conocen todas las ganancias, la conducta racional conducirá a unos resultados aparentemente colusorios. Pero a veces las ganancias de otras empresas solo pueden conocerse realizando amplios intercambios de información. Tal vez el mayor problema que plantea el mantenimiento de un resultado pactado sean las variaciones exógenas de la demanda y de los precios de los factores. Cuando los jugadores no disponen todos ellos simultáneamente de la nueva información, la
  • 25. reacción racional de una empresa podría ser interpretada por otra como una amenaza. 2. Puede surgir un exceso de capacidad en las industrias en las que es fácil entrar y hay productos diferenciados. Como las curvas de demanda de pendiente negativa de cada empresa dan lugar a unos niveles de producción cuyo coste medio es superior al coste medio mínimo, los aumentos de la producción provocan una disminución del coste medio. La diferencia entre la producción resultante y la producción correspondiente al coste medio mínimo a largoplazoes el exceso de capacidad, que puede utilizarse para disuadir a otras empresas de entrar. 3. a. Hay dos equilibrios de Nash, (100,800) y (900,600). b. Ambos directivos elegirán una estrategia orientada al segmento superior y el equilibrio resultante será (50,50), lo que generara menos beneficios a ambas partes. c. El resultado cooperativo (900,600) maximiza los beneficios conjuntos de las dos empresas. d. La empresa 1 se beneficia extraordinariamente de la cooperación. En comparación con la siguiente oportunidad mejor, la 1 se beneficia en 900-100=800, mientras que la 2 pierde 800-600=200 con cooperación. Por lo tanto, la 1 necesitaría ofrecer a la 2200 como mínimo para compensarla por la perdida. 6. a. Sí, hay dos: (1) dado que la empresa 2 elige A, la empresa 1 elige C; dado que la empresa 1 elige C, la 2 elige A. (2) Dado que la empresa 1 elige C, la 2 elige A. (2) Dado que la empresa 2 elige C, la 1 elige A; dado que la empresa 1 elige A, la 2 elige C. b. Si las dos empresas utilizan estrategias maximizan, la 1 elige el producto A y la 2 elige el producto A y la 2 elige el producto A, por lo que ambas obtienen una ganancia de -10. c. La empresa 2 elige el producto C con el fin de maximizar las ganancias en la castilla 10,20. 12. Aunque las subastas de antigüedades suelen tener componentes de valor privado, son principalmente de valor común porque hay anticuarios. Nuestro anticuario está decepcionadoen la subasta abierta al público de la ciudadcercana porque las estimaciones del valor de las antigüedades varían mucho y ha sufrido la maldición del ganador. En su ciudad natal, donde hay menos postores bien informados, la maldición del ganador no ha sido un problema. CAPITULO 14 1. Con este programa, la recta presupuestaria de los trabajadores es una línea recta en un nivel de 5.000 dólares. No hay ningún incentivo para trabajar con este nuevo
  • 26. programa. La oferta de trabajo solo es positiva cuando los salarios generan una renta superior a 10.000 dólares. 4. La demanda del trabajo viene dada por el ingreso del producto marginal, IPML = IM. OML. En el mercado competitivo, el precio es igual al ingreso marginal, por lo que IM=10. El producto marginal del trabajo es igual a la pendiente de la función de producción Q=12L – L2 . Está pendiente es igual a 12- 2L. La cantidad de trabajo que maximiza los beneficios de la empresa se encuentra donde IMPL = ω, que es el salario. Si ω=30, despejando L se obtiene 4,5 horas al día. Asimismo, si ω=60, se obtiene un valor de L de 3 horas al día. 9. La renta económica es la diferencia entre los salarios totales de todos los trabajadores ocupados menos la cantidad que habría incluido a estos trabajadores a trabajar. Los salarios totales son iguales a ω lD=1.200 ω- 10ω2. La renta total que habrían aceptado los trabajadores es el área situada debajo de la curva de oferta de trabajo hasta la cantidad de trabajo demandada correspondiente a ω. A partir de la función de oferta, sabemos que L=20 ω, o sea, ω=(1.200-10ω)/20. En ese caso, esta área es un triángulo igual a LD. ωs.1/2=(1.200-10 ω){(1.200-10ω)/20}.1/2=36.000- 600ω + 2,5 ω2= -36.000=1.800-25ω. El máximo se encuentra en el punto en el que está pendiente es igual a cero, o sea, ω=72. A un salario de 72 dólares, tienen trabajo 480 afiliados. Estos habrían estado dispuestos a trabajar por un renta total de 5.760 dólares (0,5.480.480/20). Reciben 34.560 dólares y disfrutan de unas rentas económicas de 28.800 dólares (34.560-5.760). CAPITULO 15 El valor actual descontado de los primeros 80 dólares pagados dentro de un año es VAD=80/(1+0,10)1=72,73 dólares. El valor de todos estos cupones puede hallarse de la misma forma: VAD=80{1/(1,10)1+1/(1,10)2+1/(1,10)3+1/(1,10)4+1/(1,10)5}=303,26 dolares. El valor actual del pago final de 1,000 dólares en el sexto año es 1.000/1,16=564,47 dólares. Por lo tanto, el valor actual de este bono es 303,26$ + 564,47$=867,73 dólares. Con un tipo de interés del 15 por ciento, VAD=700,49 dólares. Redefiniendo los términos, la ecuación del valor actual neto se convierte en VAN= -5- 5(1+R)-1 -1(1+R)-2 -0,5(1+R)-3 + 0,96{(1+R)-4+ (1+R)-5 + (1+R)-6 + (1+R)-7} + 0,96{(1+R)- 8 + (1+R)-9 + (1+R)-10 + (1+R)-11} + 0,96{(1+R)-12 + (1+R)-13 + (1+R)-14 + (1+R)-15} + 0,96{(1+R)-16 + (1+R)-17 + (1+R)-18 + (1+R)-19} + 0,96(1+R)-20 + 1(1+R)-20. Con un tipo de interés del 4 por ciento, el VAN se convierte en -5-4,8075 -0,9246 -0,4445 + 3,0978 + 2,6482 + 2,2637 + 1,9349 + 0,4381 + 0,4564 = -320.000 dólares. La inversión no merece la pena. a. Si compramos una botella y la vendemos dentro de t años, pagamos 100 dólares ahora y recibimos 100t0,5 cuando la vendamos. El VAN de esta inversión es VAN= 100+e-rt100t0,5. Si compramos efectivamente una botella, elegimos el valor de t que maximice el VAN. La
  • 27. condición necesaria es dVAN/dt= e-0,1t(50-t-0,5)-0,1e-0,1t.100t0,5=0. Resolviendo, se obtiene que t=5. Si conservamos la botella durante 5 años, el VAN es -100+e- 0,1.5100.50.5=35,62. Como cada botella es una buena inversión, debemos comprar las 100. b. Usted recibe 130 dólares hoy, pero pierde los 100.50, 5 dólares que recibiera vendiendo las botellas dentro de cinco años. El VAN de la oferta es VAN=130-(e (0,1) (5)) (100) (5(0,5))= -239<0. Por lo tanto, no debe venderlas. c. Si el tipo de interés baja de 10 a 5 por ciento, el cálculo del VAN es el siguiente: VAN=- 100+E-0,05t.100t0, 5. Si conservamos la botella durante 10 años, el VAN máximo es -100+e- 0.05.10.100.100, 5=91,80 dólares. 9. a. Compare la compra del automóvil con el alquiler, suponiendo que r=0,04. El valor actual del coste neto de la compra es igual a -15.000+6.000/(1+0,04)3=-9.666,02. El valor actual del coste del alquiler es igual a -3.600/(1+0,04)-3.600/(1+0,04)2-3.600/(1+0.04)3=-9.990,33. Es mejor comprar el automóvil si r=4 por ciento. b. Compare una vez más la compra con el alquiler: -15.000+6.000/(1+0,12)3=-10.729,32 en el caso de compra y -3.600/(1+0,12)-3.600/(1+0,12)2-3.600/(1+0,12)3=-8.646,6 en el caso de alquiler. Es mejor alquiler el automóvil si r=12 por ciento. c. A los consumidores les dará igual cuando el valor del coste de la compra y posterior venta del automóvil se igual al valor actual del coste del alquiler: -15.000+6.000/(1+r)3 =- 3.600/(1+r)-3.600/(1+r)2 -3.600/(1+r)3 . Esto es cierto cuando r=4,96. El lector puede resolver esta ecuación utilizando una calculadora o una hoja de cálculo o por aproximaciones sucesivas. CAPITULO 16 1. Incluso aunque las preferencias sean idénticas, la curva de contrato puede o no ser una línea recta. Es fácil mostrarlo gráficamente. Por ejemplo, cuando ambas personas tienen las funciones de utilidad U=x2 y, la relación marginal de sustitución viene dada por 2y/x. No es difícil demostrar que las RMS de las dos personas son iguales en todos los puntos de la curva de contrato y=(Y/X)/x, donde X e Y son las cantidades totales de los dos bienes. Un ejemplo en el que la curva de contrato no es una línea recta es aquel en el que las dos personas tienen rentas diferentes y un bien es inferior. 2. La relación marginal de transformación es igual al cociente entre los costes marginales de producir los dos bienes. La mayoría de las fronteras de posibilidades de producción son “combadas hacia fuera”. Sin embargo, si los dos bienes se producen con funciones de producción idénticas, la frontera de posibilidades de producción es una línea recta.
  • 28. 3. El cambio del proceso de producción de rendimientos constantes de escala por uno de rendimientos claramente crecientes no altera la forma de las isocuantas. Podemos redefinir simplemente las cantidades correspondientes a cada isocuanta de tal forma que los aumentos de la producción más que proporcionales. Partiendo de este supuesto, la relación marginal de sustitución técnica no variaría, por lo que tampoco variaría la curva de contrato de la producción. CAPITULO 17 4. a. Hasta hace poco, los clientes pensaban que los automóviles americanos eran de peor calidad. Para dar un giro a esta tendencia, las compañías americanas invirtieron en control de la calidad, mejorandolos historiales potenciales de reparaciones de sus productos. Señalaron la mejora de su calidad ofreciendo mejores garantías. b. Existe riesgo moral cuando la parte asegurada (el propietario de un automóvil americano que tiene amplia garantía) puede influir en la probabilidad o en la magnitud del acontecimiento que desencadena el pago(la reparación del automóvil). La cobertura de todas las piezas y la mano de obra en caso de problemas mecánicos reduce el incentivo para cuidar el automóvil. Por lo tanto, las garantías amplias plantean un problema de riesgo moral. CAPITULO 18 5. Es necesario saber que el valor tiene para los propietarios de viviendas la posibilidad de nadar en el rio, así como el coste marginal de reducción de la contaminación. La elección de la política depende de los beneficios y los costes marginales de reducción. Si se cobra a las empresas una tasa idéntica por los vertidos, estas los reducirán hasta el punto en el que el coste marginal de reducción sea igual a la tasa. Si esta reducción sea igual a la tasa. Si esta reducción no es suficiente para poder nadar, podría elevarse la tasa. El establecimiento de una norma solo es eficiente si las autoridades poseen una información completa sobre los costes y los beneficios marginales de la reducción de la contaminación. Por otra parte, la norma no animara a las empresas a reducir más los vertidos si surgen nuevas tecnologías para filtrarlos. Un sistema de permisos transferibles de contaminación también obliga a las autoridades a averiguar cuál es el nivel de eficiente. Una vez distribuidos los permisos, surgirán un mercado y las empresas cuyo coste de reducción de la contaminación sea más alto compraran permisos a las empresas cuyo coste sea menor. Sin embargo, a menos que se vendan inicialmente los permisos, no se obtendrá ningún ingreso. 6. a. Los beneficios se maximizan cuando el ingreso marginal es igual al coste marginal. Con un ingreso marginal constante de 20 dólares y un coste marginal de 10+2Q, Q=5.
  • 29. B. Si las abejas no acuden al manzanar, el agricultor debe pagar 10 dólares por acre por la polinización artificial. Como estaría dispuesto a pagar hasta 10 dólares al apicultor para que mantuviera cada colmena adicional, el beneficio social marginal de cada una es de 30 dólares, que es mayor que el beneficio privado marginal de 20 dólares. Igualando el beneficio social y marginal y el coste marginal, Q=10. c. El cambio más radical que haría que esta actividad fuera más eficiente seria la fusión del negocio del agricultor y el del apicultor. Esta fusión internalizaría la externalidad positiva de la polinización de las abejas. Si no se produce la fusión, el agricultor y el apicultor deberían firmar un contrato por los servicios de polinización.